MSK Flashcards

1
Q

Stress Fracture

A

Fracture resulting from the mismatch of bone strength and chronic mechanical force. They come in two flavors (A) Fatigue, and (B) Insufficiency.

How well did you know this?
1
Not at all
2
3
4
5
Perfectly
2
Q

Pathologic Fracture

A

You will sometimes hear people use this term synonymously with “Insufficiency
Fracture”. However, for the purpose of
multiple choice this term will most likely
refer to a fracture through a lvtic bone lesion.

These lytic lesions can be mets or be benign primary bone lesions (like an ABC, or Bone Cyst).

How well did you know this?
1
Not at all
2
3
4
5
Perfectly
3
Q

Open Fracture (Compound Fracture):

A

A fracture associated with an open wound. Typically these will go to the OR for reduction and washout - given the obvious risk for infection.

Tuft Fractures (finger tip fracture) with disruption o f the nail plate are considered “open” fractures - and although the typically won’t go to the OR they do get antibiotics (whereas an intact nail bed often won’t).

How well did you know this?
1
Not at all
2
3
4
5
Perfectly
4
Q

Fatigue Fracture

A

(sometime simply called a “stress fracture”).

Abnormal stress on Normal Bone.

Classic Scenario - Insane (but kinda hot) Type A Female Cross Country Runner - literally runs until her legs & feet break in half.

How well did you know this?
1
Not at all
2
3
4
5
Perfectly
5
Q

Insufficiency Fracture

A

Normal stress on Abnormal bone.

Classic Scenario - Old lady with horrible osteoporosis breaks her back (compression fracture) by walking down a few steps. She blames Obama for the fracture.

How well did you know this?
1
Not at all
2
3
4
5
Perfectly
6
Q

Phases of Fracture Healing

Overview

A

Physiology PhDs will describe 3 phases o f bone healing (Inflammatory, Reparative, and Remodeling). From a Radiologist’s perspective the most important thing to understand about this process is that around 7-14 days granulation tissue will be forming between the bone fragments. This results in an increased lucency o f the fracture site related to bone resorption.

In other words, a healing fracture will be MORE LUCENT at 7-14 days.

This explains the disclaimer cowardly Radiologists throw out when they are afraid they missed a fracture “Consider Repeat in 7-10 days, ” The idea is that in 7-10 days, you should be able to see the fracture line , if one is p re s e n t, because o f the increase in bone lucency that occurs normally in the healing process.

How well did you know this?
1
Not at all
2
3
4
5
Perfectly
7
Q

Fracture healing

overview

A

In general, bones heal in about 6-8 weeks, but is location dependent. Healing is the fastest in the phalanges (around 3 weeks), and the slowest is either the tibia or femoral neck/shaft - depending on what you read(around 2-3 months).

How well did you know this?
1
Not at all
2
3
4
5
Perfectly
8
Q

Phalanges healing

A

fast: 3 weeks

How well did you know this?
1
Not at all
2
3
4
5
Perfectly
9
Q

Tibia healing

A

slow: 10 weeks

How well did you know this?
1
Not at all
2
3
4
5
Perfectly
10
Q

Everything else healing

A

6-8 weeks

How well did you know this?
1
Not at all
2
3
4
5
Perfectly
11
Q

Delayed Union

A

fracture not healed within the expected time period (but still might). Some sources will say “twice as long as expected”

How well did you know this?
1
Not at all
2
3
4
5
Perfectly
12
Q

Non-Union

A

fracture is not going to heal without intervention. Some sources will say “6-9 months.’The classic locations are the scaphoid, anterior tibia, and lateral femoral neck.

How well did you know this?
1
Not at all
2
3
4
5
Perfectly
13
Q

Mal-Union

A

This is union in poor anatomic position (healed crooked as a politician).

How well did you know this?
1
Not at all
2
3
4
5
Perfectly
14
Q

Risk Factors For Abnormal Healing ( Delayed and Hon-Union)

Vitamin D deficiency

A
Vitamin D plays a vital role in
calcium uptake and metabolism.
Vitamin D deficiency is actually
the most common vitamin
deficiency in America
(supposedly).
How well did you know this?
1
Not at all
2
3
4
5
Perfectly
15
Q

Risk Factors For Abnormal Healing ( Delayed and Hon-Union)

Gastric bypass

A
Having your gut rewired results
in altered calcium absorption
(causes secondary
hyperparathyroid and stripping
of calcium from bones) and
therefore higher rates of nonunion.
How well did you know this?
1
Not at all
2
3
4
5
Perfectly
16
Q

Risk Factors For Abnormal Healing ( Delayed and Hon-Union)

Drugs/MEds

A

Tobacco (Smoking or Chewing)
NSA1DS
Prednisone (steroids)

How well did you know this?
1
Not at all
2
3
4
5
Perfectly
17
Q

THIS v s THAT-C om p re s s iv e Side vs T e n s ile Side:

A
  • Fractures of the Compressive side are constantly pushed back together - these do well.
  • Fractures of the Tensile side arc constantly pulled apart - these are a pain in the ass to heal.
How well did you know this?
1
Not at all
2
3
4
5
Perfectly
18
Q

Tibial Stress Fx

A

This is the most common site o f a stress fracture in young athletes.

These arc most common on the compressive side (posterior medial) in either the proximal or distal third.

Less common are the tensile side (anterior) fractures, and these favor the mid shaft. They arc bad news and don’t heal -often called “dreaded
black lines.’”

How well did you know this?
1
Not at all
2
3
4
5
Perfectly
19
Q

Femoral Stress Fx

A

Fractures along the compressive (medial) side are more common, typically seen in a younger person along the inferior femoral neck.

Fractures along the tensile (lateral) side arc more common in old people.

How well did you know this?
1
Not at all
2
3
4
5
Perfectly
20
Q

SONK (S p o n tan eo u s O s te o n e c ro s is o f th e K ne e)

overview

A

This is totally named wrong, as it is another type of insufficiency fracture. You see this in old ladies with the classic history o f “sudden pain after rising from a seated position.” Young people can get it too (much less common), usually seen after a meniscal surgery.

How well did you know this?
1
Not at all
2
3
4
5
Perfectly
21
Q

SONK (S p o n tan eo u s O s te o n e c ro s is o f th e K ne e)

key factoids

A
  • It’s an insufficiency fracture (NOT osteonecrosis) think SINK not SONK
  • Favors the medial femoral condyle (area o f maximum weight bearing)
  • Usually unilateral in an old lady without history o f trauma
  • Associated with meniscal injury
How well did you know this?
1
Not at all
2
3
4
5
Perfectly
22
Q

N a v icu la r S t r e s s F ra c tu r e

A

You see these in runners who run on hard surfaces. The thing to know is that ju st like in the wrist (scaphoid), the navicular is high risk for AVN.

How well did you know this?
1
Not at all
2
3
4
5
Perfectly
23
Q

March fracture

A

This is a metatarsal stress fracture which is fairly common. Classically seen in military recruits that are marching all day long.

How well did you know this?
1
Not at all
2
3
4
5
Perfectly
24
Q

Calcaneal Stress Fracture

A

The calcaneus is actually the most fractured tarsal bone. The fractures are usually intra-articular (75%). The stress fracture will be seen with the fracture line perpendicular to the trabecular lines.

You’ll rue the day you crossed me Trebek—ular Lines.

How well did you know this?
1
Not at all
2
3
4
5
Perfectly
25
Q

High risk stress fractures

A

Femoral neck

transverse patellar fx

anterior tibial fx (midshaft)

5th metatarsal

talus

tarsal navicular

sesamoid great toe

How well did you know this?
1
Not at all
2
3
4
5
Perfectly
26
Q

Lowe risk stress fractures

A

Femoral neck (compressive side)

longitudinal patellar fx

posterior medial tibial fracture

2nd and 3rd metatarsals

calcaneus

How well did you know this?
1
Not at all
2
3
4
5
Perfectly
27
Q

Scaphoid Fracture

overview

A

Most common carpal bone fracture.

Typical age group is an adolescents and young adults (Grandma is more likely to get a distal radial fracture with a similar mechanism — fall).

Blood flow is “retrograde” (distal to proximal). This is because the scaphoid surface is almost entirely (80%) covered with cartilage.

As such, the proximal pole most susceptible to AVN and Non- Union.

The first sign of AVN = Sclerosis (the dead bone can’t turn over /recycle)

Most common (70 %) fracture site = waist

Displacement of > 1 mm will likely get a fixation screw to pull the fragments together.

“Retrograde” (distal to proximal) via the Dorsal Carpal Branch of Radial Artery

How well did you know this?
1
Not at all
2
3
4
5
Perfectly
28
Q

Scaphoid Fracture

Trans-Scaphoid
Perilunate Dislocation

A

60% association with a scaphoid fx

How well did you know this?
1
Not at all
2
3
4
5
Perfectly
29
Q

Scaphoid Fracture

Humpback deformity

A

This deformity results from angulation of the proximal and distal fragments - in the setting of a waist fracture.

Can progress to progressive collapse and non-union.

Associated with D1S1

How well did you know this?
1
Not at all
2
3
4
5
Perfectly
30
Q

Scaphoid Fracture

Scapholunate Ligament
Disruption

A

Seen with 10-30% of distal radius and/or carpal fractures

The SL ligament is composed of 3 parts (volar, dorsal, and middle), with the dorsal band being the most important for carpal stability (opposite o f luno triquetral which is volar).

Disruption of the ligament predisposes for DISI deformity

Gap >3 mm

How well did you know this?
1
Not at all
2
3
4
5
Perfectly
31
Q

Scaphoid Fracture

AVN

A

the proximal pole is at greatest risk.

The first sign of AVN = Sclerosis (the adjacent bones will demineralize, but the avascular bone will not). Later the bone will fragment.

MRI = T1 Dark

Trivia: “Prieser Disease” is an atraumatic AVN of the scaphoid

How well did you know this?
1
Not at all
2
3
4
5
Perfectly
32
Q

SLAC and SNAC Wrists

Overview

A

Both are potential complications of trauma, with similar mechanisms.

SLAC Wrist (Scaphoid-Lunate Advanced Collapse) occurs with injury (or degeneration via CPPD) to the S-L ligament.

SNAC Wrist (Scaphoid Non-Union Advanced Collapse) occurs with a scaphoid fracture.

Just remember that the scaphoid always wants to rotate in flexion - the scaphoid-lunate ligament is the only thing holding it back. If this ligament breaks it will tilt into flexion, messing up the dynamics of the wrist. The radial scaphoid space will narrow, and the capitate will migrate proximally.

How well did you know this?
1
Not at all
2
3
4
5
Perfectly
33
Q

SLAC and SNAC wrists

Treatment

A

depends on the occupation/needs of the wrist. Wrist fusion will maximize strength, but cause a loss of motion. Proximal row carpectomy will maximize ROM, but cause a loss of strength.

How well did you know this?
1
Not at all
2
3
4
5
Perfectly
34
Q

SLAC and SNAC wrists

trivia

A
  • Radioscaphoid joint is first to develop degenerative changes
  • Capitate will migrate proximally and there will eventually be a DISI deformity
How well did you know this?
1
Not at all
2
3
4
5
Perfectly
35
Q

Scapholunate Ligament Tear

A

The Terry Thomas look (gap between the scaphoid and lunate) on plain film.

There are actually 3 parts (volar, dorsal, and middle), with the dorsal band being the most important for carpal stability. If they tear the carpals will migrate away from each other.

Predisposed for DISI deformity

How well did you know this?
1
Not at all
2
3
4
5
Perfectly
36
Q

DISI Vs VISI

Overview

A

1 imagine two people (Lunate and Scaphoid) standing on opposite sides o f a very steep hill. At the apex o f the hill is
a man named “Scapholunate Ligament” - 1 agree, it’s a strange name. His parents were probably vegetarians.

This hill is very steep, so Scapholunate Ligament has grabbed each of the people (Lunate and Scaphoid) by the
hand - he was worried they might fall. In fact, the only thing keeping these two people from tumbling down the hill is the insane grip strength o f Scapholunate Ligament (rumor has it he can close a #3 Captain o f Crush - which would certify him as an official Captain o f Crush).

By using this analogy perhaps you can infer that if you have carpal ligament disruption, the carpal bones will rotate the way they naturally want to (down the hill). The reasons for their rotational desires are complex but basically have to do with the shape of the fossa they sit on.

Just remember the scaphoid wants to flex (rock volar) and the lunate wants to extend (rock dorsal). The only thing holding them back is their ligamentous attachment to each other.

How well did you know this?
1
Not at all
2
3
4
5
Perfectly
37
Q

DISI
Dorsal Intercalated Segmental Instability

Overview

A

1 like to call this dorsiflexion instability because it helps me remember what’s going on. After a “Radial sided injury” (scapholunate side), the lunate becomes free o f the stabilizing force o f the scaphoid and rocks dorsal ly. Remember SL ligament injury is common, so this is common.

How well did you know this?
1
Not at all
2
3
4
5
Perfectly
38
Q

DISI
Dorsal Intercalated Segmental Instability

main

A

Widening of the SL angle - with dorsiflexion of the lunate.

Angle greater than 60 degrees

How well did you know this?
1
Not at all
2
3
4
5
Perfectly
39
Q

VISI (Volar Intercalated Segmental Instability)

Overview

A

I like to call this volar-flexion (palmar-flexion) instability because it helps me remember what’s going on. After a “Ulnar sided injury” (lunotriquetral side), the lunate no longer has the stabilizing force of the lunotriquetral ligament and gets ripped volar with the scaphoid (remember the scaphoid stays up late every night dreaming o f tilting volar).
Remember LT ligament injury is not common, so this is not common. It’s so uncommon in fact that if you see it - it’s probably a normal variant due to wrist laxity.

How well did you know this?
1
Not at all
2
3
4
5
Perfectly
40
Q

VISI (Volar Intercalated Segmental Instability)

main

A
Narrowing of
the SL angle - with
volar-flexion of the
lunate & scaphoid.
Angle < 30 (this acute
angle looks like a V to
me - “V” for “V")
How well did you know this?
1
Not at all
2
3
4
5
Perfectly
41
Q

Scapho-Lunate

Dissociation

A

sl wider than 3 mm

clenched fist few can worsen it

chronic sl dissocation can result in a slac wrist

How well did you know this?
1
Not at all
2
3
4
5
Perfectly
42
Q

Peri-Lunate

Dislocation

A

the lunate stays put. its the carpal bones around the lunate that move

6-% are associated with scaphoid fxs

How well did you know this?
1
Not at all
2
3
4
5
Perfectly
43
Q

Mid-Carpal

Dislocation

A

both lunate and capitate lose radial alignment

associated with triquetro lunate interosseous ligament disruption

associated with triquestral fracture

How well did you know this?
1
Not at all
2
3
4
5
Perfectly
44
Q

Lunate dislocation

A

luante move easy, others stay

it happens with a dorsal radiolunate ligament injury

most severe

How well did you know this?
1
Not at all
2
3
4
5
Perfectly
45
Q

Lunate dislocations

lesser arc

A

Pure Ligament Injury (No

Fractures)

How well did you know this?
1
Not at all
2
3
4
5
Perfectly
46
Q

Lunate dislocations

greater arc

A

Associated with fractures.
Described by saying “trans” the name of the fracture then the dislocation. Example “Trans-scaphoid, peri-lunate dislocation”

How well did you know this?
1
Not at all
2
3
4
5
Perfectly
47
Q

Lunate dislocations

space of poirier

A
Ligament free (“poor”)
area, that is a site of weakness
How well did you know this?
1
Not at all
2
3
4
5
Perfectly
48
Q

Which synovia l s p a c e s of the wrist n o rm a lly c om m u n ic a te ?

A

The answer is pisiform recess and radiocarpal joint. l can think o f two ways to ask this (1) related to fluid - the bottom line is that excessive fluid in the pisiform recess should not be considered abnormal if there is a radiocarpal effusion, and (2) that either space can be used for wrist arthrography.

How well did you know this?
1
Not at all
2
3
4
5
Perfectly
49
Q

Glenohumeral Joint and

Subacromial Bursa

A

Should NOT communicate. Implies the presence o f a

full thickness rotator cuff tear.

How well did you know this?
1
Not at all
2
3
4
5
Perfectly
50
Q

Ankle Joint and Common

(lateral) Peroneal Tendon Sheath

A

Should NOT communicate. Implies a tear o f the

calcaneofibular Ligament.

How well did you know this?
1
Not at all
2
3
4
5
Perfectly
51
Q

Achilles Tendon and

Posterior Subtalar Joint

A

Should NOT communicate. The Achilles tendon

does NOT have a true tendon sheath.

How well did you know this?
1
Not at all
2
3
4
5
Perfectly
52
Q

Pisifrom Recess and

Radiocarpal Joint

A

should normally communicate

How well did you know this?
1
Not at all
2
3
4
5
Perfectly
53
Q

Triangular Fibrocartilage Complex — TFCC

overview

A

I’ll begin by saying that this is arguably the most complex anatomy in the entire body (maybe second only to the posterior lateral comer). A detailed understanding is well beyond the scope o f the exam (probably…). Having said that, the TFC is specifically mentioned on the official study guide, so we need to at least talk about it. The TFCC functions as the primary stabilizer and shock absorber o f the distal radial ulnar joint (DRUJ). The TFCC is critical for a range o f activities (doing a pushups , punching General Zod, e tc …).

It looks crazy complicated - but you really only need to know at most 5 structures, Of the 5, the Hand Surgeon only really gives a shit about the Articular Disc and Radioulnar Ligaments.

How well did you know this?
1
Not at all
2
3
4
5
Perfectly
54
Q

Triangular Fibrocartilage Complex — TFCC

mr signal

A

“TFC Proper” (Articular Disc) will be dark on every sequence.

The ulnar attachment often looks intermediate in signal, this is normal related to loose connective tissue in the region.

The radial attachment will also have intermediate signal, but this is from the normal articular cartilage.

How well did you know this?
1
Not at all
2
3
4
5
Perfectly
55
Q

Triangular Fibrocartilage Complex — TFCC

injuries

A

“Class 1” Acute Injuries: Usually via fall onto extended wrist.

“Class 2” Chronic Degeneration: These are more common, and associated with positive ulnar variance and ulnar impaction.

Central perforations are common - and might even be “expected” on an old person.

Central Tear, with Ulnar Positive Variance and Abutment (cystic change in the lunate) -

How well did you know this?
1
Not at all
2
3
4
5
Perfectly
56
Q

Triangular Fibrocartilage Complex — TFCC

vasculature and healing

A
Similar to how the knee
meniscus has “red” and
“white” zones - the ulnar
side o f the TFC is vascular
and more likely to heal.
Radial sided injuries are
relatively avascular and
less likely to heal.
How well did you know this?
1
Not at all
2
3
4
5
Perfectly
57
Q

Triangular Fibrocartilage Complex — TFCC

vasculature and healing

A
Similar to how the knee
meniscus has “red” and
“white” zones - the ulnar
side o f the TFC is vascular
and more likely to heal.
Radial sided injuries are
relatively avascular and
less likely to heal.
How well did you know this?
1
Not at all
2
3
4
5
Perfectly
58
Q

U ln a r V a r ia n c e

overview

A

This is determined by comparing the lengths o f the ulna and distal radius.
These length differences can occur congenitally, or be acquired from impaction / fracture deformity.

How well did you know this?
1
Not at all
2
3
4
5
Perfectly
59
Q

U ln a r V a r ia n c e

positive

A

Ulnar Impaction

Syndrome

How well did you know this?
1
Not at all
2
3
4
5
Perfectly
60
Q

U ln a r V a r ia n c e

negative

A

AVN o f the Lunate

“Kienbock”

How well did you know this?
1
Not at all
2
3
4
5
Perfectly
61
Q

U ln a r Im p a c tio n Syndrome

(U ln a r A b u tm e n t):

A

Seen with positive ulnar variance.

Essentially the distal ulna smashes into the lunate, degenerating it (cystic change / geodes e tc …) and tears up the TFCC.

How well did you know this?
1
Not at all
2
3
4
5
Perfectly
62
Q

Keinbocks

A

AVN o f the lunate, seen in people in their20s-40s. The most likely testable trivia is the association with negative ulnar variance. It’s going to show signal drop out on T1.

How well did you know this?
1
Not at all
2
3
4
5
Perfectly
63
Q

Collies fracture

A
(Outward)
"Collie Dogs ” Like it Outside
Distal Metaphysis Fx
Dorsal Angulation
Old Lady Fracture
Ulnar Styloid Fx is Commonly Associated
How well did you know this?
1
Not at all
2
3
4
5
Perfectly
64
Q

Smith Fracture

A
(Inward)
Distal Metaphysis Fx
Volar Angulation
Younger Patient
Ulnar Styloid Fx is Commonly Associated
How well did you know this?
1
Not at all
2
3
4
5
Perfectly
65
Q

Barton Fx

A
(Dorsal or Volar)
Radial Rim Fx
Volar is More Common
Radial-Carpal Dislocation is the
“hallmark”
Typically Surgical (they have a
high rate of re dislocation /mal-union)
How well did you know this?
1
Not at all
2
3
4
5
Perfectly
66
Q

Radial Tilt

A
  • There is a normal volar tilt o f around 11 degrees
  • With distal radial fractures this can get fucked up
  • Most Orthopods wo n ’t accept anything past neutral
  • A TRUE lateral is necessary to measure it

How do yo u know yo u r lateral is “tru e “ ?
The volar cortex o f the pisiform overlies the central 1/3 o f the interval between the scaphoid and capitate

How well did you know this?
1
Not at all
2
3
4
5
Perfectly
67
Q

Wrist tendon extensor compartments

A

There are 6 extensor compartments (5 fingers + 1 for good luck).
First compartment (APL and EPB) are the ones affected in de Quervain’s
Third compartment has the EPL which courses beside Lister’s Tubercle.
The sixth compartment (Extensor Carpi Ulnaris) - can get an early tenosynovitis in rheumatoid arthritis.

How well did you know this?
1
Not at all
2
3
4
5
Perfectly
68
Q

Carpal Tunnel

overview

A

They could show you the carpal tunnel, but only to ask you about anatomy.
What goes through the carpal tunnel (more easily asked as “what does NOT go through ”)?
Knowing what is in (and not in) the carpal tunnel is high yield for multiple choice testing. The tunnel lies deep to the palmaris longus, and is defined by 4 bony prominences (pisiform, scaphoid tubercle, hook o f hamate, trapezium tubercle), with the transverse carpal ligament wrapping the contents in a fibrous sheath.

How well did you know this?
1
Not at all
2
3
4
5
Perfectly
69
Q

Carpal Tunnel

contents

A
  • 4 Flexor D. Profudus (FDP)
  • 4 Flexor D. Superficials (FDS)
  • 1 Flexor Pollicis Longus (FPL), and
  • 1 Median Nerve
How well did you know this?
1
Not at all
2
3
4
5
Perfectly
70
Q

Carpal Tunnel

FCR

A

The Flexor Carpi Radialis (FCR) is NOT truly in the tunnel.
The extensor tendons are on the other side o f the hand. Note that
Flexor Pollicis Longus (FPL) goes through the tunnel, but
Flexor Pollicis Brevis does not (it’s an intrinsic handle muscle).
Palmaris longus (if you have one) does NOT go through the
tunnel.

How well did you know this?
1
Not at all
2
3
4
5
Perfectly
71
Q

Carpal Tunnel

what does not go through it

A

-Flexor Carpi Radialis
-Flexor Carpi Ulnaris
-Palmaris Longus
(if you have one)
-Flexor Pollicis BREVIS

How well did you know this?
1
Not at all
2
3
4
5
Perfectly
72
Q

Carpal Tunnel Syndrome

A
  • Median Nerve Distribution (thumb-radial aspect o f 4th digit), often bilateral, and may have thenar muscle atrophy.
  • On Ultrasound, enlargement o f the nerve is the main thing to look for
  • It’s usually from repetitive trauma,
  • Trivia = Association with Dialysis. Pregnancy, DM, and HYPOthyroidism
How well did you know this?
1
Not at all
2
3
4
5
Perfectly
73
Q

Carpal Tunnel syndrome

classic findings

A

Classic Findings:
• Increased Signal in the Median Nerve
• The Nerve May Also Be Swollen or Look Smashed / Flattened
• Bowing of the Flexor Retinaculum

How well did you know this?
1
Not at all
2
3
4
5
Perfectly
74
Q

Guyon’s C an a l Syndrome

A
  • Entrapment o f the ulnar nerve as it passes through Guyon’s canal (formed by the pisiform and the hamate - and the crap that connects them).
  • Classically caused by handle bars “handle bar p a lsy.”
  • Fracture o f the hook o f the hamate can also eat on that ulnar nerve.
How well did you know this?
1
Not at all
2
3
4
5
Perfectly
75
Q

Sub-Sheath T e a r / D is lo c a tio n

A

This refers to a traumatic dislocation to the extensor carpi ulnaris (ECU - compartment 6) out o f its normal groove at the level o f the distal ulna. This dislocation / subluxation implies rupture o f the overlying sheath.

How well did you know this?
1
Not at all
2
3
4
5
Perfectly
76
Q

Sub-Sheath T e a r / D is lo c a tio n

trivia

A

the direction of the dislocation is medial

How well did you know this?
1
Not at all
2
3
4
5
Perfectly
77
Q

Tuberculous or
Nontuberculous
Mycobacterial

tenosynovitis

A

Hand and wrist are the most common tendons affected

Diffuse exuberant tenosynovitis that spares the muscles.

Usually occurs in patients who are immunocompromised.

Discrete filling defects in the fluid filled sheaths (“rice bodies”) is a classic TB finding.

How well did you know this?
1
Not at all
2
3
4
5
Perfectly
78
Q

RA

Tenosynovitis

A

Multiple Flexor Tendons or Isolated Extensor Carpi Ulnaris if early (ECU =
Compartment 6)

Tenosynovitis can present as an early RA findings (before bone findings).

How well did you know this?
1
Not at all
2
3
4
5
Perfectly
79
Q

Penetrating infection

tenosynovitis

A

Tenosynovitis of any flexor tendon is a surgical emergency as it can spread rapidly to the common flexors of the wrist.

Increased pressure in the sheath can cause necrosis of the tendons.

Patients with delayed treatment tend to do terrible

Myocobacterium Marinum is usually direct infection in a fisherman or sushi chef.

How well did you know this?
1
Not at all
2
3
4
5
Perfectly
80
Q

Overuse

Tenosynovitis

A

This is going to be classic

locations like 1 st extensor compartment for De Quervains — discussed more below.

How well did you know this?
1
Not at all
2
3
4
5
Perfectly
81
Q

De Quervain’s Tenosynovitis:

A

So called “Washer Woman’s Sprain” or “Mommy Thumb.” Occurs from repetitive activity / overuse. The classic history is “new mom - holding a baby.”

First Extensor Compartment (Extensor Pollicis Brevis and Abductor Pollicis Longus

Finkelstein Test = Pain on passive ulnar deviation.

The presence or absence of an intratendinous septum (between the EPB and APB) - tendons on the is a prognostic factor. If its absent, this will nearly always resolve with conservative treatment alone.

How well did you know this?
1
Not at all
2
3
4
5
Perfectly
82
Q

De Quervain’s Tenosynovitis

imaging

A
Ultrasound: Increased fluid
within the first extensor
tendon compartment
MRI: increased T2 signal in
the tendon sheath
How well did you know this?
1
Not at all
2
3
4
5
Perfectly
83
Q

Intersection syndrome

tenosynovitis

A

A repetitive use issue (classically seen in rowers),

Occurs where the first extensor tendons, “intersects” the second extensor compartment tendons. The result is extensor carpi radialis brevis and longus tenosynovitis.

Occurs 5 CM proximaly to listers tubercle

How well did you know this?
1
Not at all
2
3
4
5
Perfectly
84
Q

Tenosynovitis overview

A

This is an inflammation of the tendon, with increased fluid seen around the tendon. This will be shown on MRI (or US).

How well did you know this?
1
Not at all
2
3
4
5
Perfectly
85
Q

B en n e tt and Rolando

F ra c tu re s :

A
  • They are both fractures at the base o f the first metacarpal
  • The Rolando fracture is comminuted (Bennett is not)
  • Trivia: The pull o f the Abductor Pollicis Longus
How well did you know this?
1
Not at all
2
3
4
5
Perfectly
86
Q

G am e k e e p e r’s T h um b (S k ie r):

overview

A

• Avulsion fracture at the base o f the proximal first phalanx
associated with ulnar collateral ligament disruption.
• The frequently tested association is that o f a “S te n e r
Le s ion .” A Stener Lesion is when the Adductor tendon
aponeurosis gets caught in the tom edges o f the UCL.
The displaced ligament won’t heal right, and will need
surgery.
• It makes a “yo-yo” appearance on MRI - supposedly…
• Next Step - Don’t do “ stress views” that can cause a
stener. MRI is the more appropriate test.

How well did you know this?
1
Not at all
2
3
4
5
Perfectly
87
Q

G am e k e e p e r’s T h um b (S k ie r):

do not

A

DO NOT perform radiographic stress views for Gamekeepers Thumbs

this will make it a stener

How well did you know this?
1
Not at all
2
3
4
5
Perfectly
88
Q

G am e k e e p e r’s T h um b (S k ie r):

stener lesion

A

Ulnar collateral ligament is retracted and

displaced superficial to the adductor aponeurosis.

How well did you know this?
1
Not at all
2
3
4
5
Perfectly
89
Q

T rig g e r Finger:

A

The idea is the overuse /
repetitive trauma causes scarring in the flexor tendon sheath. The fancy word is “stenosing tenosynovitis.” This is most commonly shown with ultrasound. If
they should you a hand ultrasound think about this. Another common area o f “stenosing tenosynovitis” is
at the ankle specifically the flexor hallucis longus tendon around the ankle in patients with the os trigonum syndrome.

How well did you know this?
1
Not at all
2
3
4
5
Perfectly
90
Q

Elbow & Forearm

General trivia

A
  • Radial Head Fracture is most common in adults (supracondylar is most common in PEDs)
  • Sail sign - elevation o f the fat pads from a jo in t effusion. Supposedly a sign o f occult fracture. The testable trivia is (1) the posterior fat pad is more specific (posterior is positive), and (2) the posterior fat pad can appear falsely elevated (false positive) if the lateral isn’t a true 90 degree flexed lateral. “Posterior Positive, Posterior Position Dependent ”
  • Capitellum fractures are associated with posterior dislocation
How well did you know this?
1
Not at all
2
3
4
5
Perfectly
91
Q

Fo re a rm F ra c tu re s / Eponyms

Overivew

A

Forearm fractures are “ring” or “pretzel” type fractures, similar to the pelvis or mandible. Think about breaking a pretzel, it always snaps in two spots (not ju st one). So forearm fractures are often two fractures, or a fracture + dislocation.

How well did you know this?
1
Not at all
2
3
4
5
Perfectly
92
Q

Fo re a rm F ra c tu re s / Eponyms

monteggia fracture (MUGR)

A

Fracture of the proximal ulna, with anterior dislocation of the radial head.

Dislocation of the radial head follows the angulation of the Ulnar Fx.

How well did you know this?
1
Not at all
2
3
4
5
Perfectly
93
Q

Fo re a rm F ra c tu re s / Eponyms

Galeazzi fracture (MUGR)

A

Radial shaft fracture, with anterior

dislocation of the ulna at the DRUJ.

How well did you know this?
1
Not at all
2
3
4
5
Perfectly
94
Q

Essex lopresti

A

Fracture of the radial head + Anterior dislocation of the distal radial ulnar joint.

Unstable fracture - With rupture of the interosseous membrane

How well did you know this?
1
Not at all
2
3
4
5
Perfectly
95
Q

C u b ita l Tu nne l Syndrome

overview

A

There are several causes - the most common in the real world is probably repetitive valgus stress. The most common shown on multiple choice is probably an accessory anconeus.

WTF is an “Anconeus ” ? It a piece o f shit muscle that does nothing but get in the way o f an orthopedic scope. It’s normally on the lateral side the elbow. You can have an “Accessory Anconeus ” - also called an “Anconeus Epitrochlearis ’’ - on the medial side which will exert mass effect on the ulnar nerve.

How well did you know this?
1
Not at all
2
3
4
5
Perfectly
96
Q

C u b ita l Tu nne l Syndrome

anatomy trivia

A

The site where the ulnar nerve passes beneath the cubital tunnel retinaculum also known as the epicondylo-olecranon ligament or Osborne’s ligament.

How well did you know this?
1
Not at all
2
3
4
5
Perfectly
97
Q

L a te ra l E p ico n d y litis

A

(more common than medial) - seen in Tennis Players -
• Extensor Tendon Injury (classically extensor carpi radialis brevis)
• Radial Collateral Ligament Complex - Tears due to varus stress

How well did you know this?
1
Not at all
2
3
4
5
Perfectly
98
Q

M ed ia l E p ico n d y litis

A

(less common than lateral) - seen in golfers

• Common flexor tendon and ulnar nerve may enlarge from chronic injury

How well did you know this?
1
Not at all
2
3
4
5
Perfectly
99
Q

P a r tia l U ln a r C o lla te ra l L ig am e n t T e a r:

A

For the exam all you really need to know is that throwers (people who valgus overload) hurt their ulnar collateral ligament (which attaches on the medial coronoid - sublimetubercle). The ligament has three bundles, and the anterior bundle is by far the most important. If you get any images it is most likely going to be of the partial UCL tear, described as the “T sign,” with contrast material extending medial to the tubercle

How well did you know this?
1
Not at all
2
3
4
5
Perfectly
100
Q

L ittle L e a g u e r E lb ow

A

The children of insecure men who sucked at sports in high school are most
susceptible to this injury. The mechanism is repetitive micro-trauma from
endless hours of training (necessary to finally rectify the injustice which
beset their family when dad was benched senior year from the junior varsity
baseball squad).

We are talking about a repetitive
chronic injury to the medial
epicondyle. When I say injury I mean
stress fracture, avulsion, or delayed
closure of the medial epicondvlar
apophysis. This is usually associated
with UCL injury.
Children aren’t the only ones who can
fuck up their elbows pursuing the kind
of immortality that is only offered to
those worthy enough to step foot on
the field at Yankee stadium. There is a
well described “valgus overload
syndrome” seen in throwers,
consisting of a triad of lateral
compression, medial tension, and
posterior sheer. This mechanism
results in UCL injury (often anterior
band), Arthritis at the Posterior
Humerus / Ulna, and the development
of an OCD at the capitellum.
How well did you know this?
1
Not at all
2
3
4
5
Perfectly
101
Q

Epitrochlear lymphadenopathy

A

ct scratch disease

How well did you know this?
1
Not at all
2
3
4
5
Perfectly
102
Q

Dialysis elbow

A

This is the result of olecranon bursitis from constant pressure on the area, related to positioning of the arm during treatment.

How well did you know this?
1
Not at all
2
3
4
5
Perfectly
103
Q

Biceps Tear

overview

A

Tears can be partial or complete. When complete the tear typically occurs in shoulder with the tendon avulsing off the labrum (or at the level of the bicipital groove).

Common mechanism is incorrect deadlift form (while doing cross fit like an ape on cocaine). If you plan on going nuts slinging that shit around consider switching to a double over grip. If you want to use over under grips - you need strict form (keep your arms locked out dummy). There are tons of highlight reals on youtube of people tearing biceps while deadlifting - notice every single one is using an over under grip, and not maintaining straight arm technique.

How well did you know this?
1
Not at all
2
3
4
5
Perfectly
104
Q

PArtial Biceps Tear

gamesmanship

A

Partial tears often arc associated with bicipitoradial bursitis

How well did you know this?
1
Not at all
2
3
4
5
Perfectly
105
Q

Biceps Tear

gamesmanship

A

Injury to the bicep is associated with median nerve symptoms

How well did you know this?
1
Not at all
2
3
4
5
Perfectly
106
Q

Tricep rupture

A

The tricep tendon has the honorable distinction of being the LEAST common tendon in the body to rupture. Even tendinopathy is fairly uncommon relative to other nearby structures. When it does tear you should be thinking about salter harris II fractures of the olecranon - that is the classic scenario. 1 think because this is so uncommon that mimics would be more likely on the exam. So, I’d be aware of two things: (1) the normal striated appearance of the insertion at the olecranon, and (2) the common entity
of olecranon bursitis - which you should think of first if you see a bunch of fluid signal in the posterior elbow.

How well did you know this?
1
Not at all
2
3
4
5
Perfectly
107
Q

Elbow dislocation

overview

A

This is the second most common joint dislocated in the adult. The associated fractures are usually the radial head and the coronoid process.

Instability in the elbow (so called P o s te rio r R o ta ry In s ta b ility ) is described in a pattern starting in the posterior lateral comer with tearing o f the lateral UCL.

How well did you know this?
1
Not at all
2
3
4
5
Perfectly
108
Q

elbow dislocation

steps

A

Tearing of the LUCL (lateral UCL).

Partial Dislocation
Coronoid Perched on Trochlea
(LUCL + LCL + Capsule)

Dislocation
Coronoid Posterior to Humerus with a UCL Tear

How well did you know this?
1
Not at all
2
3
4
5
Perfectly
109
Q

Anterior Shoulder dislocation

A

Anterior inferior (subcoracoid) are by far the most common (like 90%).
o Hill-Sachs is on the Humerus.
o Hill-Sachs is on the posterior lateral humerus, and best seen on internal rotation view.
O Bankart - anterior inferior labrum
O Greater tuberosity avulsion fracture occurs in 10-15% o f anterior dislocations in patient’s over 40.

How well did you know this?
1
Not at all
2
3
4
5
Perfectly
110
Q

Posterior shoulder dislocation

A

Posterior Dislocation: uncommon - probably from seizure or electrocution
O Rim Sign - no overlap glenoid and humeral head
O Trough Sign - reverse Hill Sachs, impaction on anterior humerus
O “Light Bulb Sign ” - Arm may be locked in internal rotation on all views

How well did you know this?
1
Not at all
2
3
4
5
Perfectly
111
Q

Inferior dislocation

A

(lu.xatio erecta humeri) - this is an uncommon form, where the arm is sticking straight over the head. The thing to know is 60% get neurologic injury (usually the axillary nerve).

How well did you know this?
1
Not at all
2
3
4
5
Perfectly
112
Q

Hill-Sachs

A

Posterolateral humeral head
impaction fracture (anterior
dislocation)

How well did you know this?
1
Not at all
2
3
4
5
Perfectly
113
Q

Bankart

A

Anterior Glenoid Rim

anterior dislocation

How well did you know this?
1
Not at all
2
3
4
5
Perfectly
114
Q

Trough Sign

A

Anterior humeral head impaction

fracture (posterior dislocation)

How well did you know this?
1
Not at all
2
3
4
5
Perfectly
115
Q

Reverse Bankart

A

Posterior Glenoid Rim

posterior dislocation

How well did you know this?
1
Not at all
2
3
4
5
Perfectly
116
Q

Dislocation memory tool

A

1 remember that hip dislocations are posterior - from the straight leg dashboard mechanism.

Then I ju st remember that shoulders are the opposite o f that (the other one, is the other one).

Shoulder = Usually Anterior

How well did you know this?
1
Not at all
2
3
4
5
Perfectly
117
Q

Proximal humerus fx

A

This is usually in an old lady falling on an out stretched arm. Orthopods use the Neer classifications (how many parts the humerus is in ?). Three or four part fractures tend to do worse.

How well did you know this?
1
Not at all
2
3
4
5
Perfectly
118
Q

T h e Post Op S h o u ld e r (P ro s th e s is )

4 main types

A

Humeral Head Resurfacing, Hemi Arthroplasty, Total Shoulder Arthroplasty, and the Reverse Total Shoulder Arthroplasty.

How well did you know this?
1
Not at all
2
3
4
5
Perfectly
119
Q

T h e Post Op S h o u ld e r (P ro s th e s is )

reverse total shoulder

A

A conventional total shoulder mimics normal anatomy. A reserve total shoulder is the bizarro version; with a plastic cup on the humeral head and metallic sphere on the glenoid.

How well did you know this?
1
Not at all
2
3
4
5
Perfectly
120
Q

T h e Post Op S h o u ld e r (P ro s th e s is )

who gets what

A

glenoid and cuff intact - resurfacing or hemi

glenoid intact and cuff deficient - hemi or reverse

glenoid deficient cuff intact - TSA

Glenoid deficient and cuff deficient - reverse

How well did you know this?
1
Not at all
2
3
4
5
Perfectly
121
Q

T h e Post Op S h o u ld e r (P ro s th e s is )

complications/trivia

A

—Total Shoulder Most Common Complication = Loosening o f the Glenoid Component
—Total Shoulder Complication - “Anterior Escape ” - This describes anterior migration o f the humeral head after subscapularis failure.
— Reverse Total Shoulder Does NOT require an intact rotator cuff - patient rely heavily on the deltoid.
— Reverse Shoulder Complication - Posterior Acromion Fracture - from excessive deltoid tugging.

How well did you know this?
1
Not at all
2
3
4
5
Perfectly
122
Q

Impingement / Rotator Cuff Tears

overview

A

This is a high yield / confusing subject that is worth talking about in a little more detail. In
general, rotator cuff pathology is the result o f overuse activity (sports) or impingement
mechanisms. There are two types o f impingement with two major sub-divisions within those
types. Like many things in Radiology, if you get the vocabulary down, the pathology is easy to
understand.

How well did you know this?
1
Not at all
2
3
4
5
Perfectly
123
Q

Impingement / Rotator Cuff Tears

extranal overview

A

This refers to impingement o f the rotator cuff overlying the bursal surfaces (superficial surfaces) that are adjacent to the coracoacromial arch. As a reminder, the arch is made up o f the coracoid process, acromion, and coracoacromial ligament.

How well did you know this?
1
Not at all
2
3
4
5
Perfectly
124
Q

Impingement / Rotator Cuff Tears

external primary causes

A

(Abnormal Coracoacromial Arch) :
• The hooked acromion (type III Bigliani) is more associated with external impingement than the curved or flat types.
• Subacromial osteophyte formation or thickening o f the coracoacromial ligament
• Subcoracoid impingement: Impingement o f the subscapularis between the coracoid process and lesser tuberosity. This can be secondary to congenital configuration, or a configuration developed post traumatically after fracture o f the coracoid or lesser tuberosity.

How well did you know this?
1
Not at all
2
3
4
5
Perfectly
125
Q

Impingement / Rotator Cuff Tears

external secondary causes

A

(Normal Coracoacromial Arch):
• “Multidirectional Glenohumeral Instability” - resulting in micro-subluxation o f the humeral head in the glenoid, resulting in repeated micro-trauma. The important thing to know is this is typically seen in patients with generalized jo in t laxity, often involving both shoulders.

How well did you know this?
1
Not at all
2
3
4
5
Perfectly
126
Q

Impingement / Rotator Cuff Tears

internal overview

A

This refers to impingement o f the rotator cuff on the undersurface (deep surface) along the glenoid labrum and humeral head.

How well did you know this?
1
Not at all
2
3
4
5
Perfectly
127
Q

Impingement / Rotator Cuff Tears

internal posterior superior

A

This is a type o f impingement that occurs when the posterior superior rotator cuff (junction o f the supra and infraspinatus tendons) comes into contact with the posterior superior glenoid. Best seen in the ABER position, where these tendons get pinched between the labrum and greater tuberosity. This is seen in athletes who make overhead movements (throwers, tennis, swimming).

How well did you know this?
1
Not at all
2
3
4
5
Perfectly
128
Q

Impingement / Rotator Cuff Tears

internal anterior superior

A

This is internal impingement that occurs when the ann is in horizontal adduction and internal rotation. In this position, the undersurface o f the biceps and subscapularis tendon may impinge against the anterior superior glenoid rim.

How well did you know this?
1
Not at all
2
3
4
5
Perfectly
129
Q

External impingement primary

quick

A
’ Abnormal Coracoacromial Arch
— Hook Shaped (B3)
— Osteophytes
—Post Traumatic 
— Thickened Ligaments

Subacromial - F’s with a supra S
Subacoracoid - F’s with Sub S

How well did you know this?
1
Not at all
2
3
4
5
Perfectly
130
Q

External impingement secondary

quick

A

Multidirectional Instability
— Labrum Often Normal
—“Increased Glenohumeral Volume” - with injection

How well did you know this?
1
Not at all
2
3
4
5
Perfectly
131
Q

Internal impingement posterior superior

quick

A

’Throwers
— F’s with Infraspinatus (and posterior Supra)
—Posterior Superior Labrum Torn
—Cystic Change in Greater Tuberosity

How well did you know this?
1
Not at all
2
3
4
5
Perfectly
132
Q

Internal impingement anterior superior

quick

A

—Associated with Sub Scapular damage (Maybe the cause rather than the result)
—Anterior Superior Labrum Torn

How well did you know this?
1
Not at all
2
3
4
5
Perfectly
133
Q

High Yield Trivia Points on Impingement

subacromial impingement

A

most common form, resulting from attrition o f the coracoacromial arch.

Damages Supraspinatus Tendon.

How well did you know this?
1
Not at all
2
3
4
5
Perfectly
134
Q

High Yield Trivia Points on Impingement

subcoracoid impingement

A

Lesser tuberosity and coracoid do the pinching.

Damages Subscapularis (remember the coracoid is anterior - an d so is the subscapularis).

How well did you know this?
1
Not at all
2
3
4
5
Perfectly
135
Q

High Yield Trivia Points on Impingement

posterior superior “internal” impingment

A

-A th le te s who make overhead movements. Greater tuberosity and posterior inferior labrum do the pinching.

Damages Infraspinatus (and posterior fibers o f the supraspinatus).

How well did you know this?
1
Not at all
2
3
4
5
Perfectly
136
Q

Rotator Cuff Tears

overview

A

People talk about these tears as either “Bursal Sided” (meaning the top part), or “Articular Sided” (meaning the undersurface).

A tear o f the articular surface is more common (3x more) than the bursal surface. The underlying mechanism is usually degenerative, although trauma can certainly play a role.

How well did you know this?
1
Not at all
2
3
4
5
Perfectly
137
Q

Rotator Cuff Tears

most common of the 4 muscles

A

Supraspinatus - with most tears occurring at the “ c r itic a lzo n e” - 1-2 cm from the tendon footprint. This relatively avascular “critical zone” is also the most common location for Calcium Hydroxyapatite (HADD) - or “calcific tendinitis.” The Teres Minor is the least common to tear.

How well did you know this?
1
Not at all
2
3
4
5
Perfectly
138
Q

Rotator Cuff Tears

partial tear

A

A partial tear that is > 50% often results in a surgical intervention.

How well did you know this?
1
Not at all
2
3
4
5
Perfectly
139
Q

Rotator Cuff Tears

massive rotator cuff tear

A

refers to at least 2 out o f the 4 rotator cuff muscles

How well did you know this?
1
Not at all
2
3
4
5
Perfectly
140
Q

Rotator Cuff Tears

general piece of trivia

A

A final general piece o f trivia is that a tear o f the fibrous rotator cuff interval (junction between anterior fibers o f the Supraspinatus and superior fibers o f the subscapularis), is still considered a rotator cuff tear.

How well did you know this?
1
Not at all
2
3
4
5
Perfectly
141
Q

Rotator Cuff Tears

How do you know it is a full thickness tear?

A

You will have high T2 signal in the expected location o f the tendon. On T1 you will have Gad in the bursa.

How well did you know this?
1
Not at all
2
3
4
5
Perfectly
142
Q

Adhesive Capsulitis “Frozen Shoulder”

overview

A

An inflammatory condition characterized by a global decrease in motion. You can have primary types, but a multiple choice key would be a history o f trauma or surgery.

It most commonly effects the rotator cu ff interval - and that is the most likely spot they will show it. The classic look is a T1 (or non-fat sat T2) in the sagittal plane showing loss o f fa t in the rotator c a ff interval (the spot with the biceps tendon - between the Supra S,and the Sub Scap).

How well did you know this?
1
Not at all
2
3
4
5
Perfectly
143
Q

Adhesive Capsulitis “Frozen Shoulder”

buzz1

A

‘Decreased Glenohumeral Volume ” - with injection

How well did you know this?
1
Not at all
2
3
4
5
Perfectly
144
Q

Adhesive Capsulitis “Frozen Shoulder”

buzz2

A

** Remember in Multi-Directional Instability the volume was increased.

How well did you know this?
1
Not at all
2
3
4
5
Perfectly
145
Q

Adhesive Capsulitis “Frozen Shoulder”

buzz3

A

“Thickened inferior a n d Posterior Capsule ”

How well did you know this?
1
Not at all
2
3
4
5
Perfectly
146
Q

Adhesive Capsulitis “Frozen Shoulder”

buzz4

A

“Enhancement o f the Rotator C u ff Interval - Post gad

How well did you know this?
1
Not at all
2
3
4
5
Perfectly
147
Q

SLAP

overview

A

Labral tears favor the superior margin and track anterior to posterior. As this tear involves the labrum at the insertion o f the long head o f the biceps , injury to this tendon is associated and part o f the grading system (type 4).

How well did you know this?
1
Not at all
2
3
4
5
Perfectly
148
Q

SLAP

things to know

A

When the SLAP extends into the biceps anchor (type 4), the surgical management changes from a debridement to a debridement + biceps tenodesis.

The mechanism is usually an over-head movement (classic = swimmer)

People over 40 usually have associated Rotator Cuff Tears

NOT associated with Instab ility (usually)

How well did you know this?
1
Not at all
2
3
4
5
Perfectly
149
Q

SLAP

mimic 1

A

The sublabral recess

This is essentially a normal variant where you have incomplete attachment o f the labrum at 12 o’clock. The 12 o’clock position on the labrum has the shittiest blood flow - that’s why you see injury there and all these developmentvariants.

How well did you know this?
1
Not at all
2
3
4
5
Perfectly
150
Q

SLAP

mimic 2

A

The Sublabral Foramen
- This is an unattached (but present) portion o f the labrum - located at the anterior-superior labrum (1 o’clock to 3 o ’clock).

As a rule it should NOT extend below the equator (3 o’clock position).

How well did you know this?
1
Not at all
2
3
4
5
Perfectly
151
Q

SLAP

mimic 3

A

The Buford Complex - A commonly tested (and not infrequently seen) variant is the Buford Complex. It’s present in about 1% o f the general population. This consists o f an absent anterior/superior labrum (1 o’clock to 3 o ’clock), along with a thickened middle glenohumeral ligament.

How well did you know this?
1
Not at all
2
3
4
5
Perfectly
152
Q

Buford complex

A
  • Thick Middle GH Ligament

- Absent Anterior Superior Labrum

How well did you know this?
1
Not at all
2
3
4
5
Perfectly
153
Q

sublabral recess

A

Follows contour of glenoid
smooth margin
located at biceps anchor

How well did you know this?
1
Not at all
2
3
4
5
Perfectly
154
Q

B a n k a r t L e s io n s

list

A

Anterior dislocation injuries

GLAD 
Perthes
 ALPSA 
Bankart - cartilaginous
 Bankart (Osseous)
How well did you know this?
1
Not at all
2
3
4
5
Perfectly
155
Q

GLAD

A

Glenolabral Articular Disruption. It’s the most mild version, and it’s basically a superficial anterior inferior labral tear with associated articular cartilage damage (“ impaction injury with cartilage defect”). Not typically seen in patients with underlying laxity. It’s common in sports. No instability (a ren’tyou GLAD there is no instability)

How well did you know this?
1
Not at all
2
3
4
5
Perfectly
156
Q

Perthes

A

Detachment o f the anteroinferior labrum (3-6 o’clock) with medially stripped looks but intact periosteum.

Memory Aid:
-The detached
labrum sorta like a P

How well did you know this?
1
Not at all
2
3
4
5
Perfectly
157
Q

ALPSA

A

Anterior Labral Periosteal Sleeve Avulsion. Medially displaced labroligamentous complex with absence o f the labrum on the glenoid rim. Intact periosteum. It scars down to glenoid.

How well did you know this?
1
Not at all
2
3
4
5
Perfectly
158
Q

True bankart

A

Can be cartilaginous or osseous. The periosteum is disrupted. There is often an associated Hill Sach’s fracture.

How well did you know this?
1
Not at all
2
3
4
5
Perfectly
159
Q

GLAD quick

A

superficial partial labral injury

no instability

How well did you know this?
1
Not at all
2
3
4
5
Perfectly
160
Q

Perthes quick

A

Avulsed anterior labrum (only minimally displaced)

inferior GH complex still attached to periosteum

intact periosteum (lifted up)

How well did you know this?
1
Not at all
2
3
4
5
Perfectly
161
Q

ALPSA quick

A

similar to perthes but with bunhced up and medially displaced inferior GH complex

How well did you know this?
1
Not at all
2
3
4
5
Perfectly
162
Q

True bankart quick

A

torn labrum

periosteum disruption

How well did you know this?
1
Not at all
2
3
4
5
Perfectly
163
Q

P o s te rio r G len o h um e ra l In s ta b ility

overview

A

As I mentioned previously, anterior shoulder dislocations are way more common than posterior
shoulder dislocations. Therefore the Bankart, ALPSA, Perthes, e tc … are the ones you typically
think o f as the stigmata o f prior dislocation.
However, all that shit can happen in reverse with a posterior dislocation.

How well did you know this?
1
Not at all
2
3
4
5
Perfectly
164
Q

P o s te rio r G len o h um e ra l In s ta b ility

list

A

reverse osseous bankart

polpsa

bennett lesion

kims lesion

How well did you know this?
1
Not at all
2
3
4
5
Perfectly
165
Q

Reverse osseous bankart

A

A fracture o f the posterior inferior

rim o f the glenoid.

How well did you know this?
1
Not at all
2
3
4
5
Perfectly
166
Q

POLPSA

A

This is the bizarro version o f the
ALPSA, where the posterior labrum and
the posterior scapular periosteum (still intact) are stripped from the glenoid resulting in a recess that communicates with the joint space.

How well did you know this?
1
Not at all
2
3
4
5
Perfectly
167
Q

Bennett lesion

A

An extra-articular curvilinear calcification - associated with posterior labral tears tmavbe the POLPSA). It’s related to injury o f the posterior band o f the inferior glenohumeral ligament.

How well did you know this?
1
Not at all
2
3
4
5
Perfectly
168
Q

Kims lesion

A

An incompletely avulsed / flattened / mashed posteriorinferior labrum.

A key (testable) point is the glenoid cartilage and posterior labrum relationship is preserved.

How well did you know this?
1
Not at all
2
3
4
5
Perfectly
169
Q

HAGL

A

A non-Bankart lesion that is frequently tested is the HAGL (Humeral avulsion glenohumeral ligament). This is an avulsion o f the inferior glenohumeral ligament, and is most often the result o f an anterior shoulder dislocation (just like all the above bankarts). The “J Sign” occurs when the normal U-shaped inferior glenohumeral recess is retracted away from the humerus, appearing as a J.

How well did you know this?
1
Not at all
2
3
4
5
Perfectly
170
Q

S u b lu x a tio n o f th e B ic ep s Tendon

overview

A

The subscapularis attaches to the lesser tuberosity. It sends a few fibers across the bicipital groove to the greater tuberosity , which is called the “transverse ligament”. A tear o f the subscapularis opens these fibers up and allows the biceps to dislocate (usually medial). Subscapularis Tear = Medial Dislocation of the Long Head of the Biceps Tendon.

How well did you know this?
1
Not at all
2
3
4
5
Perfectly
171
Q

S u b lu x a tio n o f th e B ic ep s Tendon

subscap tendon

A

Forming portions of the “Transverse Ligament” that holds the biceps tendon in the groove

How well did you know this?
1
Not at all
2
3
4
5
Perfectly
172
Q

S u b lu x a tio n o f th e B ic ep s Tendon

quick

A

Occurs with a Tear of the Subscapularis

How well did you know this?
1
Not at all
2
3
4
5
Perfectly
173
Q

S u p ra s c a p u la r N o tch vs Sp in o g len o id N o tch

A

A cyst at the level o f the suprascapular notch will affect the supraspinatus and the infraspinatus. At the level o f the spinoglenoid notch, it will only affect the infraspinatus.

How well did you know this?
1
Not at all
2
3
4
5
Perfectly
174
Q

Q u a d rila te ra l S p a c e Syndrome

A

Compression o f the Axillary Nerve in the Quadrilateral Space (usually from fibrotic bands). They will likely show this with atrophy of the teres minor. Another classic question is to name the borders o f the quadrilateral space: Teres Minor Above, Teres Major Below, Humeral neck lateral, and Triceps medial.

How well did you know this?
1
Not at all
2
3
4
5
Perfectly
175
Q

P a rso n a g e -T u rn e r Syndrome

A

This is an idiopathic involvement o f the brachial plexus. Think about this when you see muscles affected by pathology in two or more nerve distributions (suprascapular and axillary etc..).

How well did you know this?
1
Not at all
2
3
4
5
Perfectly
176
Q

Fem ora l S h a ft F ra c tu re s

medial side

A

On the inside (medial) is the classic stress fracture location

How well did you know this?
1
Not at all
2
3
4
5
Perfectly
177
Q

Fem ora l S h a ft F ra c tu re s

lateral side

A

On the outside (lateral) is the classic bisphosphonate related fx location.
As shown in the image, you see cortical thickening (white arrow) along the lateral femur, eventually progressing into a fracture.

How well did you know this?
1
Not at all
2
3
4
5
Perfectly
178
Q

Hip fracture/dislocation

overview

A

You see these with dash board injuries. The posterior dislocation (almost always associated with a fracture as it’s driven backwards) is much more common than the anterior dislocation.

How well did you know this?
1
Not at all
2
3
4
5
Perfectly
179
Q

Hip fracture/dislocation

iliopectineal line

A

anterior

How well did you know this?
1
Not at all
2
3
4
5
Perfectly
180
Q

Hip fracture/dislocation

anterio column vs posterior column

A

The both column fracture by definition divides the ilium proximal to the hip joint, so you have no articular surface o f the hip attached to the axial skeleton (that’s a problem).

How well did you know this?
1
Not at all
2
3
4
5
Perfectly
181
Q

Hip fracture/dislocation

hip fx leading to avn

A

The location o f the fracture may predispose to AVN. It’s important to remember that, since the femoral head gets vascular flow from the circumflex femorals, a displaced intracapsular fracture could disrupt this blood supply - leading to AVN.

Testable Point: Degree o f fracture displacement corresponds with risk o f AVN.

How well did you know this?
1
Not at all
2
3
4
5
Perfectly
182
Q

Hip fracture/dislocation

corona mortis

A

The anastomosis o f the inferior epigastric and obturator vessels sometimes rides on the superior pubic ramus. During a lateral dissection - sometimes used to repair a hip fracture - this can be injured. 1 talk about this more in the vascular chapter.

How well did you know this?
1
Not at all
2
3
4
5
Perfectly
183
Q

Hip fracture/dislocation

hip fx leading to avn

A

The location o f the fracture may predispose to AVN. It’s important to remember that, since the femoral head gets vascular flow from the circumflex femorals, a displaced intracapsular fracture could disrupt this blood supply - leading to AVN.

Testable Point: Degree o f fracture displacement corresponds with risk o f AVN.

How well did you know this?
1
Not at all
2
3
4
5
Perfectly
184
Q

Avulsion In jury :

Hip overview

A

This is seen more in kids than adults. Adult bones are stronger than their tendons. In kids it’s the other way around. One pearl is that if you see an isolated “ avulsion” of the lesser trochanter in a seemingly mild trauma / injury in an adult - query a pathologic fracture. Now, to discuss what 1 believe to be one o f the highest yield topics in MSK, “where d id the avulsion come from? ”
The easiest way to show this is a plain film pelvis (or MRI) with a tug/avulsion injury to one o f the muscular attachment sites. The question will most likely be “what attaches there? ” or “which muscle got avulsed? ”

How well did you know this?
1
Not at all
2
3
4
5
Perfectly
185
Q

Muscle that originates

iliac crest

A

abdominal muscles

How well did you know this?
1
Not at all
2
3
4
5
Perfectly
186
Q

Muscle that originates

asis

A

sartorius TFL

How well did you know this?
1
Not at all
2
3
4
5
Perfectly
187
Q

Muscle that originates

AIIS

A

rectus femoris

How well did you know this?
1
Not at all
2
3
4
5
Perfectly
188
Q

Muscle that originates

greater trochanter

A

Gluteal muscles

How well did you know this?
1
Not at all
2
3
4
5
Perfectly
189
Q

Muscle that originates

lesser trochanter

A

iliopsoas

How well did you know this?
1
Not at all
2
3
4
5
Perfectly
190
Q

Muscle that originates

ischial tuberosity

A

hamstrings

How well did you know this?
1
Not at all
2
3
4
5
Perfectly
191
Q

Muscle that originates

pubic symphysis

A

adductor group

How well did you know this?
1
Not at all
2
3
4
5
Perfectly
192
Q

Snapping Hip Syndrome

steps

A

Clinical Eval for the “E x tern a l Type”
(IT band “snapping” over the Greater Trochanter)

This is to evaluate for the “In tra-Articular Type”
Looking for Hip Degen / Loose Bodies, Etc Radiographs?

IF NO Degen NEXT S T EP = Ultrasound
This is to look for the “In te rn a l Type”
Look for Dynamic “Snapping” o f the
Iliopsoas over the Uiopectineal eminence or femoral head
*This has to he shown with a CINE - because the finding is a dynamic moving o f a tendon I f yo u see a hip ultrasound fo r snapping - this is what they are going fo r

IF US Negative NEXT S T EP = MRI Arthrogram
This is to evaluate for the “Intra-Articular Type” … Again
This time looking for Labral Tears

How well did you know this?
1
Not at all
2
3
4
5
Perfectly
193
Q

Snapping Hip Syndrome

trivia

A

clinical sensation they have to tell you the pt feels snapping

How well did you know this?
1
Not at all
2
3
4
5
Perfectly
194
Q

Snapping Hip Syndrome

types

A
  • External (most common) = Iliotibial Band over Greater Trochanter
  • Internal = Iliopsoas over Uiopectineal eminence or femoral head
  • Intra-Articular = Labral tears / jo in t bodies
How well did you know this?
1
Not at all
2
3
4
5
Perfectly
195
Q

Snapping Hip Syndrome

types

A
  • External (most common) = Iliotibial Band over Greater Trochanter
  • Internal = Iliopsoas over Uiopectineal eminence or femoral head
  • Intra-Articular = Labral tears / jo in t bodies
How well did you know this?
1
Not at all
2
3
4
5
Perfectly
196
Q

IT Band Syndrome

A

This is a repetitive stress syndrome seen most classically in runners. The key finding is fluid on both sides of the IT band, extending posterior and lateral.

Fluid in the joint does not exclude the diagnosis, but for the purpose of multiple choice if you see fluid around the band and none in the joint you can be fairly certain this is the pathology the question writer is after.

How well did you know this?
1
Not at all
2
3
4
5
Perfectly
197
Q

Hip Labrum

what to know

A

1: Anterior-Superior Tears (white arrows) are by far the most common.
2: Paralabral Cysts (black arrow) are associated with tears and likely a hint that a tear is present.
3: Just like a shoulder intra-articular contrast will increase your sensitivity.

How well did you know this?
1
Not at all
2
3
4
5
Perfectly
198
Q

Ilio p so a s Bursa

overview

A
  • Largest bursa o f the entire body.
  • Communicates with the joint in 15% o f the population
  • Seen Anterior to the hip
  • Trivia: The illiospsoas tendon runs anterior to the labrum on axial and can mimic a tear.
How well did you know this?
1
Not at all
2
3
4
5
Perfectly
199
Q

Ilio p so a s Bursa

gamesmanship

A

A fluid signal “mass” with anterior to the femur (adjacent to the psoas tendon) at the level of the ischial tuberosity is likely Iliopsoas Bursitis

How well did you know this?
1
Not at all
2
3
4
5
Perfectly
200
Q

F em o ro a c e ta b u la r Im p in g em e n t (FAI):

overview

A

This is a syndrome of painful hip movement.
It’s based on hip / femoral deformities, and honestly might be total BS. Supposedly it can lead to early
degenerative changes. There arc two described subtypes: (A) Cam and (B) Pincher (technically there is a
mixed type - but I anticipate multiple choice to make it more black and white).

How well did you know this?
1
Not at all
2
3
4
5
Perfectly
201
Q

F em o ro a c e ta b u la r Im p in g em e n t (FAI):

memory aid

A

I remember that the femoral one (cam-type)
is more common in men because the femoral
head kinda looks like a penis.
Be honest, you were thinking that too.

How well did you know this?
1
Not at all
2
3
4
5
Perfectly
202
Q

F em o ro a c e ta b u la r Im p in g em e n t (FAI):

cam type

A

This is an osseous “bump” along the femoral head-neck junction.

How well did you know this?
1
Not at all
2
3
4
5
Perfectly
203
Q

F em o ro a c e ta b u la r Im p in g em e n t (FAI):

pincer type

A

Whereas the CAM type is a deformity of the femur, the pincer type represents a deformity of
the acetabulum. Whereas the CAM type is more common in a young athletic male, the pincer is more
common in a middle aged woman (insert sexist joke here).

How well did you know this?
1
Not at all
2
3
4
5
Perfectly
204
Q

F em o ro a c e ta b u la r Im p in g em e n t (FAI):

coxa profunda

A

acetabulu projects medial to the ilioschial line

How well did you know this?
1
Not at all
2
3
4
5
Perfectly
205
Q

F em o ro a c e ta b u la r Im p in g em e n t (FAI):

acetabular protrusion

A

Femur projects medial to the ilioischial line

How well did you know this?
1
Not at all
2
3
4
5
Perfectly
206
Q

F em o ro a c e ta b u la r Im p in g em e n t (FAI):

os acetabuli

A

This is an unfused secondary >=> ossification center. It’s actually normal in kids (should fuse by adult hood). It has several testable associations including FAI and Labral Tears

How well did you know this?
1
Not at all
2
3
4
5
Perfectly
207
Q

FAI

classic way to ask

A

The most classic way to show or ask this is the so-called
“cross over sign”, where the acetabulum is malformed -
causing the posterior lip to “Cross over” the anterior lip. A
Key point is that the coccyx needs to be centered at the
symphysis pubis to even evaluate this (rotation fucks things
up).
The other associated finding) s) of the pincer subtype worth
knowing are the acetabular over coverage buzzwords
(Coxa Profunda and Protrusio), and the Ischial Spine Sign:

How well did you know this?
1
Not at all
2
3
4
5
Perfectly
208
Q

Total Hip Arthroplasty

bone remodeling/stress shielding

A

The stress is transferred through the metallic stem, so the bone around it is not loaded. Orthopods call this “Wolff’s Law’ - where the unloaded bone just gets resorbed.
Happens more with uncemented arthroplasty. To some degree this is a normal finding - but when advanced can predispose to fracture.

How well did you know this?
1
Not at all
2
3
4
5
Perfectly
209
Q

Total Hip Arthroplasty

asymptomatic complications

A
  • Stress Shielding

* Aggressive Granulomatosis

How well did you know this?
1
Not at all
2
3
4
5
Perfectly
210
Q

Total Hip Arthroplasty

proximal stress shielding

A

Proximal stress shielding -greater trochanter bone resorption

How well did you know this?
1
Not at all
2
3
4
5
Perfectly
211
Q

Total Hip Arthroplasty

distal stress loading

A

Distal stress loading: cortical thickening & pedestral (around the bottom)
“Zone 4“

How well did you know this?
1
Not at all
2
3
4
5
Perfectly
212
Q

Total Hip Arthroplasty

heterotopic ossifications

A

This is very common (15-50%). It’s usually asymptomatic. The trivia regarding multiple choice tests is that “hip stiffness” is the most common complaint. Also in Ank Spon patients, because they are so prone to heterotopic ossifications, they sometimes give them low dose prophylactic radiation prior to THA.

How well did you know this?
1
Not at all
2
3
4
5
Perfectly
213
Q

Total Hip Arthroplasty

aseptic loosening

A

This is the most common indication for revision. The criteria on x-ray is > 2 mm at the interface (suggestive). If you see migration of the component, you can call it (migration includes varus tilting o f the femora! stem).

How well did you know this?
1
Not at all
2
3
4
5
Perfectly
214
Q

Total Hip Arthroplasty

subsidence

A

Basically an arthroplasty
that is sliding downward. This is a described reason for early failure of THA. You see this most often in
arthroplasty implants without a collar.

Greater than 1 cm along the femoral component, or
progression after 2 years are indications of loosening.

How well did you know this?
1
Not at all
2
3
4
5
Perfectly
215
Q

Total Hip Arthroplasty

wear patterns

A

It is normal to have a little bit of thinning in the area of weight bearing - this is called “Creep.” It is not normal to see wear along the superior lateral aspect.
• Wear = Pathologic
• Creep = Normal

How well did you know this?
1
Not at all
2
3
4
5
Perfectly
216
Q

Total Hip Arthroplasty

particle diseaase

A

(Aggressive Granulomatosis): Any component of the device that sheds will
cause an inflammatory response. The more wear that occurs the more particles — wear is the
primary underlying factor. Macrophages will try and eat the particles and spew enzymes all over
the place. This process can cause progressive lytic focal regions around the replacement and joint
effusions.

How well did you know this?
1
Not at all
2
3
4
5
Perfectly
217
Q

Total Hip Arthroplasty

things to know about particle disease

A

Most commonly seen in non-cementcd hips

Tends to occur 1 -5 years after surgery — “late complication”

X-ray shows “smooth” endosteal scalloping (distinguishes from infection)

Aseptic - ESR & CRP will be normal

Produces no secondary bone response — no sclerosis

Can be seen around screw holes (particles are transmitted around screws)

How well did you know this?
1
Not at all
2
3
4
5
Perfectly
218
Q

Total Hip Arthroplasty

particle disease steps

A

wear > particle disease > osteolysis

How well did you know this?
1
Not at all
2
3
4
5
Perfectly
219
Q

Sacrum

overview

A

You can get fractures o f the sacrum in the setting o f trauma, but if you get shown or asked anything about the sacrum it’s going to be either (a) SI degenerative change - discussed later, (b) unilateral SI infection, (c) a chordoma - discussed later, (d) sacral agenesis, or (e) an insufficiency fracture. Out o f these 5 things, the insufficiency fracture is probably the most likely.

How well did you know this?
1
Not at all
2
3
4
5
Perfectly
220
Q

Sacrum

insuffieciency fx

A

The most common cause is postmenopausal osteoporosis. You can also see this in patients with renal failure, patients with RA, pelvic radiation, mechanical changes after hip arthroplasty, or extended steroid use. They are often (usually) occult on plain films.

How well did you know this?
1
Not at all
2
3
4
5
Perfectly
221
Q

Sacrum

insufficiency fx imaging

A

They will have to show this either with a bone V
scan, or MRI. The classic “Honda Sign” from the
“H” -shaped appearance is probably the most likely presentation on a mult,pie choice test.

How well did you know this?
1
Not at all
2
3
4
5
Perfectly
222
Q

Segond F ra c tu re :

A

This is a fracture o f the Lateral Tibial Plateau (icommon distractor is medial tibia). The thing to
know is that it is associated with ACL tear (75%), and occurs with internal rotation.

How well did you know this?
1
Not at all
2
3
4
5
Perfectly
223
Q

reverse segond fx

A

This is a fracture of the Medial Tibial Plateau. The thing to know is that it is associated with a PCL tear, and occurs with external rotation. There is also an associated medial meniscus injury.

How well did you know this?
1
Not at all
2
3
4
5
Perfectly
224
Q

arcuate sign

A

This is an avulsion o f proximal fibula (insertion of arcuate ligament complex). The thing to know is that 90% are associated with cruciate ligament injury (usually PCL)

How well did you know this?
1
Not at all
2
3
4
5
Perfectly
225
Q

deep intercondular notch sign

A

This is a depression of the lateral femoral condyle (terminal sulcus) that occurs secondary to an impaction injury. This is associated with ACL tears.

How well did you know this?
1
Not at all
2
3
4
5
Perfectly
226
Q

ACL anatomy

A

Composed of two bundles (anteromedial & posterolateral). The tibial attachment is thicker then the femoral attachment. Both the ACL and PCL are intra-articular and extrasynovial.

How well did you know this?
1
Not at all
2
3
4
5
Perfectly
227
Q

PCL anatomy

A

The strongest ligament in the knee (you don’t want a posterior dislocation of your knee resulting in dissection of your popliteal artery).

How well did you know this?
1
Not at all
2
3
4
5
Perfectly
228
Q

MCL anatomy

A

The MCL fibers arc laced into the joint capsule at the level of the joint, with connection to the medial meniscus. Unlike the ACL and PCL, the MCL is an extra-articular structure.

How well did you know this?
1
Not at all
2
3
4
5
Perfectly
229
Q

Conjoint tendon anatomy

A

Formed by the biceps femoris

tendon and the LCL.

How well did you know this?
1
Not at all
2
3
4
5
Perfectly
230
Q

ACL and PCL overview

A
ACL & PCL are extrasynovial and intraarticular.
The synovium folds around the
ligaments. This is why a tom ACL won’t
heal on its own (usually).
The ligament can be tom even if the
synovium is intact - this is why the
“taunt” angle of the ligament is a key
feature of integrity - more on that later
How well did you know this?
1
Not at all
2
3
4
5
Perfectly
231
Q

IT band inserts on

A

Gerdys tubercle

How well did you know this?
1
Not at all
2
3
4
5
Perfectly
232
Q

M a g ic A n g le Phenomeno n

A

The PCL and Patellar tendon may have foci o f intermediate signal intensity on sagittal images with short echo time (TE) sequences where the tendon forms an angle o f 55 degrees with the main magnetic field (magic angle p h en omen o n ).

This will NOT be seen on T2 sequences (with long TE). This phenomenon is reduced at higher field strengths due to greater shortening o f T2 relaxation times.

How well did you know this?
1
Not at all
2
3
4
5
Perfectly
233
Q

M a g ic A n g le Phenomeno n

quick

A

You sec it on short TE sequences (Tl, PD, GRE). It goes away on T2

How well did you know this?
1
Not at all
2
3
4
5
Perfectly
234
Q

ACL T e a r:

overview

A
  • Associated with Segond Fracture (lateral tibial plateau) and tibial spine avulsion
  • ACL Angle lesser than Blumensaat’s Line
  • O’donoghue’s Unhappy Triad: ACL Tear, MCL Tear, Medial Meniscal Tear
  • Classic Kissing Contusion Pattern: The lateral femoral condyle (sulcus terminals) bangs into the posterior lateral tibial plateau. This is 95% specific in adults.
  • Anterior Drawer Sign = Ortho Physical Exam Finding suggesting ACL Tear.
How well did you know this?
1
Not at all
2
3
4
5
Perfectly
235
Q

ACL Mucoid D e g e n e ra tio n

A

This can mimic acute or chronic partial tear of the ACL. There will be no secondary signs of injury (contusion etc..). It predisposes to ACL ganglion cysts, and they are usually seen together. The T2/STIR buzzword is “celery stalk” because o f the striated look. T h eT l buzzword is “drumstick” because it looks like a
drum stick.

How well did you know this?
1
Not at all
2
3
4
5
Perfectly
236
Q

ACL repair

A

ACL can be repaired with two primary methods. Method 1: Using the middle one-third of the
patellar tendon, with the patella bone plug attached to one end and tibial bone plug attached at the
other. Method 2: Using a graft made of the semitendinosus or gracilis tendon, or both. The graft is
then attached with all sorts of screws, bolts, etc… There is a lower reported morbidity related to
harvest site using this method.

How well did you know this?
1
Not at all
2
3
4
5
Perfectly
237
Q

ACL Graft evaluation

tibial tunnel

A

Should parallel the roof of the femoral intercondylar notch. Too Steep = Impinged by femur on extension. Too Flat = Lax & won’t provide stability. Too Far Anterior (“Intersection with Blumensaat line”) = Can lead to pinching at the anterior inferior Normal intercondylar root. Buzzword Root Impingement.

How well did you know this?
1
Not at all
2
3
4
5
Perfectly
238
Q

ACL Graft evaluation

femoral tunnel

A

Supposedly the primary factor for maintaining length and tension during range of motion. This is referred to as “maintained isometry.”

How well did you know this?
1
Not at all
2
3
4
5
Perfectly
239
Q

ACL Graft evaluation

arthrofibrosis

A

Can be focal or diffuse (focal is more common). The focal form is the so called “Cyclops” lesion - so named because of its arthroscopic appearance. It’s gonna be a low signal
mass-like scar in Hoffa’s fat pad. It’s bad because it limits extension.

Buzzword “palpable audible clunk”

Seen around 16 weeks - it obviously won’t occur immediately post op because you have to build up your scar.

How well did you know this?
1
Not at all
2
3
4
5
Perfectly
240
Q

ACL Graft evaluation

cyclops lesion

A

scar associated with ventral graft

How well did you know this?
1
Not at all
2
3
4
5
Perfectly
241
Q

ACL Graft evaluation

trivia 1

A

The graft is most susceptible to tear in the remodeling process (4-8 months post op).

How well did you know this?
1
Not at all
2
3
4
5
Perfectly
242
Q

ACL Graft evaluation

trivia 2

A

Other signs of graft tear: grossly high T2 signal (some is ok), fiber discontinuity, uncovering of the posterior horn of the lateral meniscus (secondary sign), anterior tibial translation (secondary sign).

How well did you know this?
1
Not at all
2
3
4
5
Perfectly
243
Q

ACL Graft evaluation

Graft teat

A

Usually Ortho can just pull on his fucking leg is see if the graft is trashed (anterior drawer sign). For imaging, the simple way to understand this: “flat angle = tear. The ACL should parallel the roof of the intercondylar notch. If the angle becomes flat, a tear is likely.

How well did you know this?
1
Not at all
2
3
4
5
Perfectly
244
Q

Femoral Tunnel =

Tibial Tunnel =

A

1 maintains isometry

2 roof impingement

How well did you know this?
1
Not at all
2
3
4
5
Perfectly
245
Q

o s te rio r L a te ra l C o rn e r (PLC):

A

The most complicated anatomy in the entire body. My God this posterior lateral corner! Just think about the LCL, the IT band, the biceps femoris, and the popliteus tendon. The most likely way to show this on a single image (multiple choice style) is edema in the fibular head.

Who cares? Missed PLC injury is a very common cause o f ACL reconstruction failure.

How well did you know this?
1
Not at all
2
3
4
5
Perfectly
246
Q

PCL Tear overview

A

The posterior collateral ligament is the strongest ligament in the knee. A tear is actually uncommon, it’s more likely to stretch and appear thickened ( > 7 mm). PCL tears should make you think about posterior dislocation as the mechanism o f injury..

How well did you know this?
1
Not at all
2
3
4
5
Perfectly
247
Q

PCL tear next step/association

A

If you see a PCL Tear - look at the popliteal flow void. If the knee dislocated posterior, a dreaded consequence is vascular compromise. Depending on the wording o f the question they might need a run-off (watch your back).

How well did you know this?
1
Not at all
2
3
4
5
Perfectly
248
Q

Meniscal anatomy

A

The meniscus is “C shaped”, thick along the periphery and thin centrally.

Medial meniscus is thicker posteriorly.

Lateral meniscus has equal thickness between anterior and posterior portion.

How well did you know this?
1
Not at all
2
3
4
5
Perfectly
249
Q

Meniscal healing

A

The Peripheral “Red Zone” is vascular and might heal.

The Central “white zone” is avascular and will not heal. The blood supply comes from the geniculate arteries (which enter peripherally).

How well did you know this?
1
Not at all
2
3
4
5
Perfectly
250
Q

Meniscal tears

overview

A

As stated, the peripheral meniscus (red zone) has better vasculature than the inner 2/3s (white zone) and
might heal on its own. In general, you can group tears based on their general direction (as seen on a
sagittal section MRI - i.e. the triangles and bowties) - as either vertical (top-to-bottom) or horizontal
(front-to-back). You can then sub-group them depending how they look on subsequent sections.

How well did you know this?
1
Not at all
2
3
4
5
Perfectly
251
Q

Meniscal tears

radial tear

A
- Bad because they
cause “loss of hoop
strength.”
- Can lead to extrusion,
early OA etc..
Radial Tearing
Cuts the Circular
Hoop Fibers that
Hold the Meniscus
Together
How well did you know this?
1
Not at all
2
3
4
5
Perfectly
252
Q

Meniscal tears

flap tear (parrot beak)

A

Radial Tear that Changes
Direction into the
longitudinal direction

How well did you know this?
1
Not at all
2
3
4
5
Perfectly
253
Q

Meniscal tears

longitudinal tear

A
Can be vertical or
horizontal (or mixed
oblique patterns)
- Defined by a long
extension in the axial
direction
- Vertical Types can flip
(bucket- handle)
How well did you know this?
1
Not at all
2
3
4
5
Perfectly
254
Q

Meniscal tears

horizontal cleavage tear

A
- Pure cleavage tears extend to
the apex
- Associated with Meniscal Cysts
- Most common in posterior horn
of the medial meniscus
How well did you know this?
1
Not at all
2
3
4
5
Perfectly
255
Q

Meniscal tears

radial tear classic signs

A

usually 2 are present

truncated triangle

cleft (most reliable)

ghost or absent triangle

How well did you know this?
1
Not at all
2
3
4
5
Perfectly
256
Q

Meniscal tears

bucket handle tear

A

This is a tom meniscus (usually medial -
80%) vertical longitudinal sub-type, that flips
medially to lie anterior to the PCL.

How well did you know this?
1
Not at all
2
3
4
5
Perfectly
257
Q

Meniscal tears

bucket handle tear gamesmanship

A

Most likely shown as the
classic Aunt Minnie appearance of a “double PCL.”

Can also be shown as “not enough bowties,” the opposite of the “too
many bowties” look of a discoid meniscus.

Only 1 bowtie -
instead o f the normal 2. The m iddle o f the second bowtie is flipped medially.

How well did you know this?
1
Not at all
2
3
4
5
Perfectly
258
Q

Meniscal tears

bucket handle trivia

A

The appearance of a double PCL can
only occur in the setting of an intact ACL,
otherwise it won’t flip that way. Just know it
sorta indirectly proves the ACL is intact (I
can just see some knucklehead asking that).

How well did you know this?
1
Not at all
2
3
4
5
Perfectly
259
Q

Meniscal tears

discoid meniscus gamesmanship 1

A

“Pediatric Patient with Meniscal

Tear”.

How well did you know this?
1
Not at all
2
3
4
5
Perfectly
260
Q

Meniscal tears

discoid meniscus gamesmanship 2

A

If shown on sagittal
they have to show you 3 or more “bow ties” /
double triangles.

How well did you know this?
1
Not at all
2
3
4
5
Perfectly
261
Q

Meniscal tears

discoid meniscus bow ties

A

Normal Meniscus will have 2 bowtie shapes in the
sagittal plane - assuming 3mm slices with 1mm
gap-
Discoid Meniscus will have 3 or more bowties

How well did you know this?
1
Not at all
2
3
4
5
Perfectly
262
Q

Meniscal tears

discoid meniscus gamesmanship 3

A
If
shown on coronal they
need to show you a
meniscus stretching
into the notch.
How well did you know this?
1
Not at all
2
3
4
5
Perfectly
263
Q

Discoid meniscus trivia

A

There are three types, with the most rare
and most prone to injury being the Wrisberg
Variant.

How well did you know this?
1
Not at all
2
3
4
5
Perfectly
264
Q

Discoid meniscus trivia

A

There are three types, with the most rare
and most prone to injury being the Wrisberg
Variant.

How well did you know this?
1
Not at all
2
3
4
5
Perfectly
265
Q

Meniscal cysts

A

Most often seen near the lateral meniscus and are often associated with horizontal cleavage tears.

How well did you know this?
1
Not at all
2
3
4
5
Perfectly
266
Q

Bakers Cyst

A

Occurs between the
semimembranosus and the
MEDIAL head o f the gastroc

How well did you know this?
1
Not at all
2
3
4
5
Perfectly
267
Q

M e n is c o c a p s u la r S e p a ra tio n

A

This is a rare (in real life - maybe not on exams) injury. The idea is that the deepest layer o f the MCL complex (capsular ligament) is relatively weak and is the first to tear. This deep tearing may result in the separation o f the meniscus and the MCL. I’ve never seen it occur in isolation (theoretically it can). The important things to remember are probably (1) it happens more with proximal MCL tears, and (2) this is a serious injury — requires immobilization or surgery.

How well did you know this?
1
Not at all
2
3
4
5
Perfectly
268
Q

M en is c a l Ossicle

A

This is a focal ossification o f the posterior horn o f the medial meniscus, that can be secondary to trauma or simply developmental. They are often associated with radial root tears.

How well did you know this?
1
Not at all
2
3
4
5
Perfectly
269
Q

Meniscofemoral ligaments

A

There are 2 (Wrisberg, Humphry) which can be mimics o f meniscal tears. Wrisberg is in the back ( “humping H um p h ry”). You could also remember that “H” comes before “W” in the alphabet.

How well did you know this?
1
Not at all
2
3
4
5
Perfectly
270
Q

Meniscal flounce

A

This an uncommon finding o f a “ ru ffled ”
appearance o f the meniscus that mimics a tear.
It’s NOT associated with an increased incidence o f
tear - but can look like one, if you don’t have any
idea what one looks like.

How well did you know this?
1
Not at all
2
3
4
5
Perfectly
271
Q

Patella Dislocation:

overview

A

Dislocation o f the patella is usually lateral
because o f the shape o f the patella and femur.
The contusion pattern is classic.

How well did you know this?
1
Not at all
2
3
4
5
Perfectly
272
Q

Patella Dislocation:

points

A
• It’s Lateral
• Contusion Pattern - Classic
•Associated tear o f the MPFL
(medial patellar femoral ligament)
•Associated with “Trochlear Dysplasia ” - the trochlea is too flat.
How well did you know this?
1
Not at all
2
3
4
5
Perfectly
273
Q

P a te lla A lta / Baja

A

The patella will move up or down in certain traumatic situations. If the quadricep tendonmtears you will get unopposed pull from the
patellar tendon resulting in a low patella (Baja). If the patella tendon tears you will get unopposed quadriceps tendon pull resulting in
a high patella (Alta).

The “classic” association with patellar tendon tear (Alta) is SLE, (also can see in elderly,trauma, athletics, or RA).

How well did you know this?
1
Not at all
2
3
4
5
Perfectly
274
Q

bilateral patellar rupture is a buzzword for

A

chronic steroids

How well did you know this?
1
Not at all
2
3
4
5
Perfectly
275
Q

Prepatellar

Bursitis

A

fluid superficial to the patella

How well did you know this?
1
Not at all
2
3
4
5
Perfectly
276
Q

fat impingement syndrome

A

high T2 signal in hoffas fat ingerior to the patella

How well did you know this?
1
Not at all
2
3
4
5
Perfectly
277
Q

jumpers knee

A

high t2 signal + thickening of the inferior patella

How well did you know this?
1
Not at all
2
3
4
5
Perfectly
278
Q

Tib ia l P la te a u F ra c tu re

A

This injury most commonly occurs from axial loading (falling and landing on a straight leg). The lateral plateau is way more common than the medial. If you see medial, it’s usually with lateral. Some dude named Schatzker managed to get the classification system named after him, o f which type 2 is the most common (split and depressed lateral plateau).

How well did you know this?
1
Not at all
2
3
4
5
Perfectly
279
Q

pilon fx (tibial plafond)

A

This injury also most commonly occurs from axial loading, with the talus being driven into the tibial plafond. The fracture is characterized by comminution and articular
impaction. About 75% o f the time you are going to have fracture o f the distal
fibula.

How well did you know this?
1
Not at all
2
3
4
5
Perfectly
280
Q

tibial shaft fx

A

This is the most common long bone fracture. It was also listed as the most highly tested subject in orthopedic OITE exam (with regard to trauma), over the last 8 years. Apparently there are a bunch o f ways to put a nail or plate in it. It doesn’t seem like it could be that high yield for the CORE compared to other fractures with French or Latin sounding names. I will point out that the tibia is one o f the slowest healing bones in the body (10 weeks).

How well did you know this?
1
Not at all
2
3
4
5
Perfectly
281
Q

Tillaux fx

overview

A

This a Salter-Harris 3,
through the anterolateral aspect o f the distal tibial
epiphysis.

How well did you know this?
1
Not at all
2
3
4
5
Perfectly
282
Q

Tillaux fx

trivia 1

A

This pattern requires an open physis along the
lateral distal tibia. This is why you see this fracture
pattern in the window between the start o f medial
physis fusion and the complete fusion o f the lateral
physis (lateral physis typically closes around 12-15).

How well did you know this?
1
Not at all
2
3
4
5
Perfectly
283
Q

Tillaux fx

trivia 2

A

The distal tibial growth plate closes from

medial to lateral (medial first).

How well did you know this?
1
Not at all
2
3
4
5
Perfectly
284
Q

Triplane fx

A

This is a Salter-Harris
4, with a vertical component through the
epiphysis, horizontal component through the
physis, and oblique through the metaphysis.

*The addition o f the fracture plane in the
posterior distal tibial metaphysis (coronal plane)
distinguishes this from the Tillaux.

How well did you know this?
1
Not at all
2
3
4
5
Perfectly
285
Q

Maisonneuve fx

A

This is an unstable fracture involving the medial tibial malleolus and/or disruption of the distal tibiofibular syndesmosis.

The most common way to show this is to first show you the ankle with the widened mortis, and “next step? ” get you to ask for the proximal fibula - which will show the fracture o f the proximal fibular shaft.

This fracture pattern is unique as the forces begin distally in the tibiotalar jo in t and then ride up the syndesmosis to the proximal fibula.

How well did you know this?
1
Not at all
2
3
4
5
Perfectly
286
Q

Maisonneuve fx

trivia

A

does not extend ino the hindfoot

How well did you know this?
1
Not at all
2
3
4
5
Perfectly
287
Q

Maisonneuve

wide medial malleolus

A

(+/- Medial Malleolus Fracture)
— Distal Tibiofibular syndesmosis
+/- Deltoid Lig Injury^

How well did you know this?
1
Not at all
2
3
4
5
Perfectly
288
Q

maisonneuve

proximal fibular fx

A

— From upwardforce extension

( “the rippin an d the tearin ”) via the syndesmosis

How well did you know this?
1
Not at all
2
3
4
5
Perfectly
289
Q

Casanova fx

overview

A

If you see bilateral calcaneal fractures, you should “next s te p ? ” look
at the spine (T12-L2) for a compression or burst fracture. These tend to occur in axial loading
patterns (possibly from jumping out a window to avoid an angry husband).

How well did you know this?
1
Not at all
2
3
4
5
Perfectly
290
Q

Casanova fx

trivia

A
  • Peroneal tendons can become entrapped with lateral calcaneal fractures.
  • Calcaneal fractures are the most common (60%) Tarsal Bone Fx
  • Fractures o f the calcaneus are either extra-articular or intra-articular - depends on subtalar jo in t involvement. Intra-articular fractures will have a fracture line through the “critical angle o f Gissane ”
How well did you know this?
1
Not at all
2
3
4
5
Perfectly
291
Q

Casanova fx

bohlers angle

A
The line
drawn between the anterior
and posterior borders o f the
calcaneus on a lateral view.
An angle less than 20
degrees, is concerning for a
fracture.
How well did you know this?
1
Not at all
2
3
4
5
Perfectly
292
Q

Casanova fx

critical angle of gissane

A

“More Flat” (More than 130)
Equals Depression of the Posterior Facet

normal is 95-105

How well did you know this?
1
Not at all
2
3
4
5
Perfectly
293
Q

Stress fx of the 5th metatarsal

A

This is considered a high risk fracture (hard to heal).

How well did you know this?
1
Not at all
2
3
4
5
Perfectly
294
Q

Jones Fx

A

This is a fracture
at the base o f the fifth metatarsal, 1.5cm
distal to the tuberosity. These are placed
in a non-weight bearing cast (may
require internal fixation- because o f risk
o f non-union.

How well did you know this?
1
Not at all
2
3
4
5
Perfectly
295
Q

Avulson fracture of the 5th metatarsal

A
This is more common than a jones
fracture. The classic history is a dancer.
It may be secondary to tug from the
lateral cord of the plantar
aponeurosis or peroneus brevis (this is
controversial).
How well did you know this?
1
Not at all
2
3
4
5
Perfectly
296
Q

Painful Os Peroneus S yndrom e (POPS)

A

*Os Peroneus (accessory ossicle) is within the Peroneus LONGUS
•This ossicle is seen in about 10% of gen pop
•Stress reaction and pain can progress to tendon disruption = POPS

How well did you know this?
1
Not at all
2
3
4
5
Perfectly
297
Q

Painful Os Peroneus S yndrom e (POPS)

key mr findings

A

Edema in the os peroneus just before the peroneus longus tendon enters the cuboid tunnel

How well did you know this?
1
Not at all
2
3
4
5
Perfectly
298
Q

L is fra n c In jury:

overview

A

This is the most common dislocation of the
foot. The Lisfranc joint is the articulation o f the tarsals and
metatarsal bases. This joint is recessed creating a “keystone”
locking mechanism, and would make a good place to amputate if
you were a surgeon assisting in the Napoleonic invasion o f
Russia. The Lisfranc ligament connects the medial cuneiform to
the 2nd metatarsal base on the plantar aspect.

How well did you know this?
1
Not at all
2
3
4
5
Perfectly
299
Q

L is fra n c In jury:

keys

A
  • Can’t exclude it on a non-weight bearing film
  • Associated fractures are most common at the base o f the 2nd MT - “Fleck Sign ”
  • Fracture non-union and post traumatic arthritis are gonna occur if you miss it (plus a lawsuit).
How well did you know this?
1
Not at all
2
3
4
5
Perfectly
300
Q

L is fra n c In jury:

fleck sign

A

This is a small bony fragment
in the Lisfranc Space (between 1st MT and 2nd
MT) - that is associated with an avulsion o f the
LF ligament.

How well did you know this?
1
Not at all
2
3
4
5
Perfectly
301
Q

L is fra n c In jury:

mechanism

A

Extreme Plantar Flexion +

Axial Load

How well did you know this?
1
Not at all
2
3
4
5
Perfectly
302
Q

L is fra n c In jury:

ligaments

A

3 Ligaments make up the
complex between the
medial cuneiform and
2nd MT.

plantar band is the strongest

How well did you know this?
1
Not at all
2
3
4
5
Perfectly
303
Q

Achilles tendon anatomy trivia

A

This is the largest tendon in the body. It represents the fused tendons of the gastrocnemius and the soleus muscles.

It does NOT have a tendon sheath, so it cannot have a tenosynovitis (fluid in the sheath).

Instead inflammatory change around the tendon is referred to as a “paratendinitis.”

304
Q

Lower leg anatomy

posteromedial

A

Tom - tibialis posterio
dick - flexor digitorum longus
harry - flexor hallucis longus

tibial nerve funs by harry

305
Q

lower leg anatomy

posterolateral

A

peroneus longus is more lateral than peroneus brevis

306
Q

The Mythical M a s te r K n o t off H e n ry

A

This has a funny sounding name, therefore it’s high yield. This is where Dick (FDL) crosses over Harry (FHL) at the medial ankle.
Harry starts out lateral relative to Dick. They cross at the “master knot” and then
Harry (white) ends up medial on it’s way to the big toe (Harry = Hallucis).

307
Q

The Mythical M a s te r K n o t off H e n ry

memory

A

its a harry dick

308
Q

L ig am e n to u s In ju ry

A

The highest yield fact is that the anterior talofibular ligament is the weakest ligament and the most frequently injured (usually from inversion).

309
Q

Posteriot tibial tendon injury/dysfunction

A

This results in a progressive
flat foot deformity, as the PTT is the primary stabilizer o f the longitudinal arch. When
chronic, the tear is most common behind the medial malleolus (this is where the most
friction is). When acute, the tear is most common at the insertion into the navicular bone.
Acute Flat Arch should make you think o f PTT tear.

310
Q

Posteriot tibial tendon injury/dysfunction

trivia

A

You will also have a hindfoot valgus deformity (from unopposed
peroneal brevis action). The other point o f trivia to know is that
the spring ligament is a secondary supporter o f the arch (it holds
up the talar head), and it will thicken and degenerate without the
help o f the PTT. Don’t get it twisted though, the spring ligament is
very thick and strong and almost never ruptures in a foot/ankle
trauma.

311
Q

I say acute flat foot and you say

A

posterior tibial tendon injury

312
Q

Sinus T a rs i Syndrome

overview

A

The space between the lateral talus and calcaneus. The sinus tarsi is not ju st a joint space. It is an important source o f proprioception and balance. Fucking it up has consequences (if
your goal is to make prima ballerina assoluta).

The “syndrome” is caused by hemorrhage or inflammation o f the synovial recess with or without tears o f the associated ligaments (talocalcaneal ligaments, inferior extensor retinaculum). There are associations with rheumatologic disorders and abnormal loading (flat foot in the setting o f a posterior tibial tendon tear).

313
Q

Sinus T a rs i Syndrome

acute

A

Never make this diagnosis in the setting o f acute trauma

314
Q

Sinus T a rs i Syndrome

mri

A

MRI finding is obliteration of fat in the sinus tarsi space, and replacement with scar.

315
Q

P la n ta r F a s c iitis

overview

A

This is an inflammation o f the fascia secondary to either repetitive trauma
(overuse via endless rounding on fat diabetic, smokers as a medicine intern),
abnormal mechanics (pes cavus, etc), or arthritis (Reiters, etc…).
The pain is localized to the origin o f the plantar fascia, and worsened by
dorsiflexion o f the toes. This is usually a clinical diagnosis.

316
Q

P la n ta r F a s c iitis

anatomy

A
As a rapid anatomy review, the
plantar fascia consists o f 3 bands with
the central / lateral part normally
thicker than the medial part the
thinnest.
317
Q

P la n ta r F a s c iitis

xray

A

Plain film might show heel spurs (which are not
specific), but could be a hint. A bone scan may
show increased tracer in the region o f the
calcaneus (from periosteal inflammation).

318
Q

P la n ta r F a s c iitis

mri

A
a thickened fascia
(> 4m m ), most
often the central
band
with increased T2
signal, most
significant near
its insertion at the
heel.
319
Q

classic flat foot progression

A

Posterior Tibial Tendon Goes Out

Spring ligament out

Sinus Tarsi gets ja c k e d (including those little proprioception nerves that are in it)

You start walking like an idiot, heel striking over and over again

Plantar fasciitis

320
Q

Split peroneus brevis

A

You can see longitudinal splits in the peroneus in people with inversion injuries. The history is usually “chronic ankle pain” .

The tendon will be C shaped or boomerang shaped with central thinning and partial envelopment o f the peroneus longus. Alternatively, there may be 3 instead o f 2 tendons. The tear occurs at the lateral malleolus.

There is a strong (80%) association with lateral ligament injury.

321
Q

A n te ro la te ra l Im p in g em e n t Syndrome:

A

Injury to the anterior talofibular ligaments and tibiofibular ligaments (usually from an inversion injury) can cause lateral instability, and chronic synovial inflammation.

You can eventually produce a “mass” o f hypertrophic synovial tissue in the lateral gutter.

The MRI finding is a “meniscoid mass” in the lateral gutter o f the ankle, which is a balled up scar (T1 and T2 dark).

322
Q

T a rs a l Tu nne l Syndrome:

A

Pain in the distribution o f the posterior tibial nerve (first 3 toes) from compression as it passes through the tarsal tunnel (behind the medial malleolus).

It’s usually unilateral (unlike carpal tunnel which is usually bilateral), unusually “ idiopathic” although pes planus (hindfoot valgus) can predispose by tightening the retinaculum.

Having said that, any mass lesion (ganglion cysts, neurogenic tumors, varicosities, lipomas, severe tenosynovitis, and accessory muscles) can cause compression o f the nerve in the
tunnel.

323
Q

T a rs a l Tu nne l Syndrome:

anatomy and atrophy

A

Tarsal tunnel is a covered by the flexor
retinaculum (arrows) and includes tom, dick,
harry the posterior tibial artery and nerve.
You can see atrophy of multiple foot muscles
(not just minimi as seen with “Baxter”).

324
Q

Mortons neuroma

overview

A

Soft tissue mass (tear drop shaped) shown between the 3rd and 4th
metatarsal heads (third intermetatarsal space) is most likely a Morton’s Neuroma (especially on
multiple choice tests). The proposed pathology results from compression / entrapment of the plantar
digital nerve in this location by the intermetatarsal ligament. Over time this results in thickening and
development of perineural fibrosis.

325
Q

Mortons neuroma

mulders sign

A

is a physical exam (a sonographic sign) where you squeeze the patients
foot and reproduce the pain (or see the scar pop out under ultrasound).

326
Q

Mortons neuroma

trivia

A

Morton’s Neuroma is NOT a Neuroma (a tumor).

It’s a scar.

327
Q

Mortons neuroma

classic look

A

It is a scar, so it’s gonna be dark on T 1 and T2

(usually). It is tear drop shaped and projects downward.

328
Q

Mortons neuroma

ddx

A

The reason is that your primary differential is intermetatarsal bursitis - which will extend above
the transverse ligament, be fluid signal, and have a more cystic look. Small bursa in this location can be
normal as long as the stay smaller than 3mm.

329
Q

Mortons neuroma

mri stuff intermetarsal bursitis

A

bursitis T2 bright

bursitis above the ligament (dumbbell shaped)

Mortons is T1 Dark Below the Plantar Ligament

330
Q

Haglunds syndrome/deformity

A

This is also called the “Mulholland deformity” for the purpose of fucking with you. Depending on what you read there are either 3 or 4 classic features.

The deformity is the “bump.” The “syndrome” is the bursitis and Achilles tendon thickening. They call this thing the “pump bumps,” because wearing highheeled shoes is supposedly a
predisposing factor

331
Q

Haglunds syndrome/deformity

classic features

A

• Retro-Achilles bursitis, /
• Retrocalcaneal bursitis,
• Thickening of the distal Achilles tendon
(insertional portion)
• Calcaneal Bony Prominence “prominent posterior superior os calcis”

332
Q

Haglunds syndrome/deformity

classic features

A

• Retro-Achilles bursitis, /
• Retrocalcaneal bursitis,
• Thickening of the distal Achilles tendon
(insertional portion)
• Calcaneal Bony Prominence “prominent posterior superior os calcis”

333
Q

Os Trigonum Syndrome

overview

A

The idea is that the Os Trigonum (accessory
ossicle) puts the smash on the FHL (“Harry”)
during extreme ankle flexion — toe pointing
shit (“Pointe technique”) that ballet dancers
do … or other repetitive micro trauma.

334
Q

Os Trigonum Syndrome

classic findings

A
(1) “Stenosing” tenosynovitis /
collection of fluid around the FHL, and
(2) edema within the Os Trigonum and
across the synchondrosis between the
Os and the Posterior Talus.
335
Q

Os Trigonum Syndrome

buzzword

A

ballet dancer

336
Q

Os Trigonum Syndrome

mri

A

Axial T2 - Fluid Around the FHL
Edema in the Os and Posterior

sag t2 - edema in the o and posterior talus

337
Q

what is this synchonrosis

A

This is a joint that has
essentially no
movement and is
lined with cartilage.

338
Q

A c h ille s Tendon In jury

A

Acute rupture is usually obvious with a fluid filled gap.
The gap size will determine treatment (big gaps need
surgery). The tear is usually 4 cm above the calcaneal
insertion and the classic history is an unconditioned middle
aged fake athlete (“weekend warrior”) with acute pain and
loss of the ability to plantar flex.
Without a large gap these things can be very hard to tell
apart from a Xanthoma (both can just look like a very
thick tendon). There are a few differences that can be used
to differentiate - per my chart.

339
Q

Achilles Tendon Tear

partial / small gap

A
Thick Tendon
(> 7mm)

unilateral

associatdd with being a fake athlete

step 1 trivia - fluoroquinolones

340
Q

Xanthoma

A
Thick Tendon
(> 7mm)

bilateral usually

associated with having very high cholesterol

341
Q

Plantaris rupture

A

(“Tennis Leg”): This is usually presented as the classic trick: “Achilles
tendon ruptured but can still plantar flex.” Remember not everyone
has this tendon (it’s absent in 10% of the population). The classic look
on MRI is focal fluid collection between the solcus and the medial
head of the gastrocnemius. There is an association with ACL tears.

342
Q

Avulsions of the Calcaneal Tuberosity:

A

This is sort of an Aunt Minnie with the back of the bone totally ripped
off via the Achilles. The classic association is diabetes. When you see
this you have to think diabetes.

343
Q

O s teo p en ia

A

This ju st means increased lucency o f bones. Although this is most
commonly caused by osteoporosis, that is not always the case.

344
Q

osteomalacia

A

This is a soft bone from excessive uncalcified osteoid. This is
typically related to vitamin D issues (either renal causes, liver causes, or other misc
causes). It generally looks ju st like diffuse osteopenia. For the purpose o f multiple choice
you should think about 4 things: Ill-defined trabeculae, Ill-defined corticomedullary
junction, bowing, and “ Loosers Zones.”

345
Q

Looser Zones

A

These things are wide lucent
bands that transverse bone at right angles to
the cortex. These things can happen in lots
o f different locations - but the classic two
are the fem o ra l neck and the pubic rami.
Typically there is sclerosis surrounding the
lucency. You should think two things:
osteomalacia and rickets. Less common is
01. The other piece o f trivia is to
understand they are a type o f insufficiency
fracture.

— Raise your suspicion for this shit when
you see (1) Symmetric Findings
(2) The “90 Degree” to the cortex line

346
Q

Osteopo ro sis

overview

A

The idea is that you have low bone density. Bone density peaks around
30 and then decreases. It decreases faster in women during menopause. The imaging
findings are a thin sharp cortex, prominent trabecular bars, lucent metaphyseal bands, and
spotty lucencies.

347
Q

Osteopo ro sis

causes

A

Age is the big one. Medications (steroids, heparin, dilantin), Endocrine issues
(cushings, hyperthryoidism), Anorexia, and Osteogenesis Imperfecta.

348
Q

Osteopo ro sis

complications

A

Fractures - Most commonly o f the spine (2nd most common is the hip,
3rd most common is the wrist).

349
Q

DEXA

things to know

A
  • T score = Density relative to young adult
  • T score defines osteopenia vs osteoporosis
  • T score > -1.0 = Normal, -1.0 to -2.5 = Osteopenia, < -2.5 Osteoporosis
  • Z score = Density relative to age-matched control “to Za Zame Age”
  • False negative / positive (see below)
350
Q

DEXA

fals positive

A
  • Absent Normal Structures: Status post laminectomy
351
Q

DEXA

false negative

A
  • Including excessive Osteophytes, dermal calcifications, or metal
  • Including too much o f the femoral shaft when doing a hip - can elevate the
    number as the shaft normally has denser bone.
  • Compression Fx in the area measured
352
Q

FRAX

trivia

A

• FRAX calculates fracture risk at a 10 year probability
• FRAX adds “value” by helping to identify the subset o f osteopenic patients who are at a
higher risk for fracture - and might benefit from pharmacologic intervention
• FRAX is NOT supposed to be used in patient who have already been placed on meds for
osteoporosis. The entire point o f the FRAX is to make big pharma more money… 1
mean help identify those who would benefit most from pharmacologic intervention -
those already on meds do n ’t need identified.
• FRAX is applicable for men and women
• FRAX is recommended to calculate 10 year fracture risk in patients with a T-Score
between -1 and -2.5.
• Some guidelines suggest pharmacologic intervention for patients with a FRAX
calculated 10 year hip fracture risk o f > 3% or major fracture risk o f > 20%

353
Q

Reflex Sympathetic Dystrophy (RSD)

overview

A

Also called “Complex Regional Pain Syndrome” — which makes it sound like some
Rheumatology Psycho-somatic bullshit (i.e. fibromyalgia).
Also called “Sudeck Atrophy” - which makes it sound serious - like some incurable
neurodegenerative death sentence.
The classic clinical vignette is a history o f trauma or infection.

354
Q

Reflex Sympathetic Dystrophy (RSD)

radiographs

A

On plain film, it can cause severe osteopenia (like disuse osteopenia). Some people say it looks
like unilateral RA, with preserved joint spaces. Hand and shoulder are the most common sites
o f involvement.

355
Q

Reflex Sympathetic Dystrophy (RSD)

bone scan

A

It’s one o f the many causes o f a 3 phase hot bone scan. In fact, intra-articular uptake o f tracer on
bone scan is typically seen (on multiple choice) in patients with RSD (secondary to the increased
vascularity o f the synovial membrane), and this is somewhat characteristic.

356
Q

Transient ostoporosis of the hip

A

For the purpose o f multiple choice tests, by far you
should expect to see the female in the 3’” trimester o f pregnancy with involvement o f
the left hip. Having said that, it’s actually more common in men in whom it’s usually
bilateral. The joint space should remain normal. It’s se lf limiting (hence the word
transient) and resolves in a few months. Plain film shows osteopenia, MRI shows Edema,
Bone scan shows increased uptake focally.

357
Q

Regional migratory osteoporosis

A

This is an idiopathic disorder which has a very classic
history o f pain in a joint, which gets better and then shows up in another joint. It’s
associated with osteoporosis - which is also self-limiting. It’s more common in men.

358
Q

Hip Edema - Strategy Session — This vs That - Transient Osteoporosis vs A V N vs Fx

radiograph

A

On radiograph, transient osteoporosis and AVN
look totally different. Transient Osteoporosis is
super lucent - so lucent that sometimes you can
barely see the femoral head. AVN on the other
hand, will have patchy areas o f sclerosis.

359
Q

Hip Edema - Strategy Session — This vs That - Transient Osteoporosis vs A V N vs Fx

MRI

A

On MRI, the story is different. These things can look similar. They both have edema on STIR, and
they arc both are dark on T1. The difference is that AVN should be shown with a serpiginous dark
line (double line if you are lucky) - that represents infarct core. Joint effusions can be seen in both -
so this isn’t helpful.

360
Q

Hip Edema - Strategy Session — This vs That - Transient Osteoporosis vs A V N vs Fx

insufficiency fx

A

Now - if these assholes want to take it to the twilight zone, they can add “insufficiency fracture” to
the list o f distractors. This is really a dirty trick as both Transient Osteoporosis and AVN are
susceptible to this. The distinction is that this fracture line should be less serpiginous and instead
parallel the subchondral bone of the femoral head.

361
Q

Osteoporotic Compression Fracture

A

Super Common. On MR you want to see
a “bandlike” fracture line - which is typically T l dark (T2 is more variable). The nondeformed
portions o f the vertebral body should have nonnal signal.

362
Q

Neoplastic Compression

Fracture:

A
Most vertebral mets
don’t result in compression fracture
until nearly the entire vertebral
body is replaced with tumor. If you
see abnormal marrow signal (not
band like) with involvement o f the
posterior margin you should think
about cancer.
363
Q

What is this “Abnormal Marrow Signal” ?

A

Normally (in an adult) the marrow of the spine is
fatty - so it should be Tl bright. The internal control
is an adjacent nonnal disc (not a desiccated disk).
If you see dark stuff - it might just be red marrow.
BUT if it is darker than the adjacent (nonnal) disc,
you have to assume that it’s a bad thing.

364
Q

Neoplastic compression fx

next step

A

Look at the rest o f the spine - mets are often multiple.

365
Q

THIS VS THAT - Osteoporotic Fx vs Neoplastic Fx

A

If I was going to show a Neoplastic vs Osteoporotic Fracture case, this is how 1 would do it.
I would have 2 sequences, a Tl and a STIR. The STIR would be positive (bright on both).
The Osteoporotic Fracture would have a T l dark line. The Neoplastic Fracture would be
diffusely low Tl signal and blobby - and I would stick a few lesions in other vertebral bodies.

366
Q

THIS VS THAT - Osteoporotic Fx vs Neoplastic Fx

A

If I was going to show a Neoplastic vs Osteoporotic Fracture case, this is how 1 would do it.
I would have 2 sequences, a Tl and a STIR. The STIR would be positive (bright on both).
The Osteoporotic Fracture would have a T l dark line. The Neoplastic Fracture would be
diffusely low Tl signal and blobby - and I would stick a few lesions in other vertebral bodies.

367
Q

O s te o c h o n d r itis D is s e c a n s (OCD)

overview

A

The new terminology is actually to call these “OCLs” (the “L” is for
Lesion). This a spectrum o f aseptic separation o f an osteochondral
fragment which can lead to gradual fragmentation o f the articular surface
and secondary OA. Most o f the time it is secondary to trauma, although it
could also be secondary to AVN.

368
Q

O s te o c h o n d r itis D is s e c a n s (OCD)

where it happens

A

Classic locations include the femoral condyle (most

common site in the knee), patella, talus, and capitellum.

369
Q

O s te o c h o n d r itis D is s e c a n s (OCD

staging

A
  • Stage 1: Stable - Covered by intact cartilage, Intact with Host Bone
  • Stage 2: Stable on Probing, Partially not intact with host bone.
  • Stage 3: Unstable on Probing, Complete discontinuity o f lesion.
  • Stage 4: Dislocated fragment
370
Q

O s te o c h o n d r itis D is s e c a n s (OCD

treatment/who cares

A

If the fragment is unstable you can get secondary OA. You want to
look for high T2 signal undercutting the fragment from the bone to call it unstable
(edema can force a false positive). Thus, the absence o f high T2 signal at the bone fragment
interface is a good indicator o f osseous bridging and stability. Granulation tissue at the
interface (which will enhance with Gd), does not mean it’s stable.

371
Q

O s te o c h o n d r itis D is s e c a n s (OCD

trivia

A
Most common in
males under 18
Most common in
the lateral aspect
of the medial
femoral condyle
372
Q

Osteochondritis
Dissecans of the capitellum

quick

A

Capitellum o f the dom in a n t arm in throwers

Anterior convex margin o f the capitellum. Unstable lesions are characterized by high signal fluid th a t encircles the osteochondral fragment on T2W image

Can lead to intra-articular
loose bodies

S lightly older patients
(12-16 years)

373
Q

Panners disease (capitellum)

quick

A

Also in th e ca p ite llum o f
throwers

Entire Capitellum
is abnormal in signal d f lP
(low T 1, high T2)

Loose b o d y fo rmation is
NOT seen (usually)

5 to 10 years old
“Peter Pan wanted to stay
yo u n g ”

374
Q

pseudo-lesion of the capitellum

A

Posterior capitellum

A coronal image through the
po s te rio r ca p ite llum can mimic
a defect. This o c cu rs because
the mos t p o s te rio r portion o f
the ca pitellum has an a b ru p t
slope.
375
Q

O s t e o c h o n d r o s e s

A

These are a group o f conditions (usually seen in childhood) that are characterized by
involvement o f the epiphysis, or apophysis with findings o f collapse, sclerosis, and
fragmentation - suggesting osteonecrosis.

376
Q

Kohlers

A

Tarsal Navicular Boys 4-6. Treatment is not surgical.

377
Q

freiberg infraction

A

Second Metatarsal Head
Adolescent Girls
- can lead to secondary OA

378
Q

severs

A

Calcaneal Apophysis
Some say this is a normal
“growing pain”

379
Q

panners

A

Capitellum Kid 5-10 “Thrower”;

does not have loose bodies.

380
Q

perthes (LCP)

A

Femoral Head White kid; 4-8.

381
Q

Keinbock

A

Carpal Lunate Associated with negative ulnar

variance. Seen in adults 20-40.

382
Q

Scheuermann

A

Thoracic Spine
Causes kyphosis. 3 adjacent levels
with wedging, plus a thoracic kyphosis
of > 40 degrees (normal 20-40)

383
Q

Osgood Schlatter disease

A

Tibial Tubercle
Adolescents (10-15) who jump and
kick. Need Fragmentation + Soft
Tissue Swelling.

384
Q

Sindig larsen johansson

A

Inferior Patella Adolescents (10-15) who jump.

385
Q

Infection overview

A

With regard to osteomyelitis, radiographs will be normal for 7-10 days. Essentially,
osteomyelitis can have any appearance, occur in any location, and occur at any age. Children
have hematogenous spread usually hitting the long bones (metaphysis). Adults are more
likely to have direct spread (in diabetic).

However, you can
have hematogenous
spread in certain
situations as well (IV
Drugs).
386
Q

Infection

general rule

A

Septic joints more common in adults. Osteomyelitis more common in kids.

387
Q

Infection

classic look

A

Hallmarks are destruction o f bone and periosteal new bone formation.

388
Q

Osteomyelitis in Spine

A

Osteomyelitis in Spine = IV Drug User

389
Q

Osteomyelitis in Spine with Kyphosis (Gibbus Deformity) =

A

TB

390
Q

Unilateral SI joint infection

A

Unilateral SI joint = IV Drug User

391
Q

Psoas Muscle Abscess =

A

TB

392
Q

Sequestrum

A

Piece o f necrotic bone

surround by granulation tissue

393
Q

Involucrum

A

Thick sheath o f

periosteal bone around sequestrum

394
Q

Cloaca

A

Defect in the periosteum

(bone skin) caused by infection

395
Q

sinus tract

A

A channel from the
bone to the skin (lined with
granulation tissue).

396
Q

Chronic Osteomyelitis

timeframe

A

This is defined as osteomyelitis lasting longer than 6 weeks.
Some trivia worth knowing:

397
Q

Chronic Osteomyelitis

trivia

A

• Draining sinus tracts are a risk factor for squamous cell CA
• Most specific sign o f active chronic osteomyelitis is the presence o f a
sequestrum (best shown with computed tomography)
• MRI diagnosis o f healed osteomyelitis is based on the return o f normal fatty marrow

398
Q

A cu te B a c te r ia l O s t e om y e litis

overview

A

can be thought o f in three different categories:
1) hematogenous seeding (most common in child), 2) contiguous spread, and 3) direct
inoculation o f the bone either from surgery or trauma.

399
Q

A cu te B a c te r ia l O s t e om y e litis

hematogenous

A

Acute hematogenous osteomyelitis has a predilection for the long bones o f the body,
specifically the metaphysis, which has the best blood flow and allows for spreading o f the
infection via small channels in the bone that lead to the subperiosteal space.

400
Q

A cu te B a c te r ia l O s t e om y e litis

Age < 1 month

A

Multi-centric involvement, often
with joint involvement. Bone scan often negative
(75%) at this age

401
Q

A cu te B a c te r ia l O s t e om y e litis

Age < 18 months = Spread to epiphysis through
blood

A

Spread to epiphysis through
blood

In the slightly older baby (<18 months) these vessels from the metaphysis to the epiphysis
atrophy and the growth plate stops the spread (although spread can still occur). This creates
a “ septic tank” effect. This same thing happens with certain cancers (leukemia); the garbage
gets stuck in the septic tank (metaphysis). Once the growth plates fuse, this obstruction is
no longer present.

402
Q

A cu te B a c te r ia l O s t e om y e litis

Age 2-16 years

A

Trans-physeal vessels are closed

primary focus is metaphysis

403
Q

MRI fin d in g s o f osteomyelitis

A
Low signal in the bone marrow on Tl imaging adjacent to an
ulcer or cellulitis is diagnostic.
STIR - High Signal in
Bone Adjacent to Ulcer
(more sensitive sign)
T1 - Corresponding Low Signal
(more specific sign)
404
Q

The Ghost Sign: Neuropathic Bone vs Osteomyelitis in a Neuropathic Bone

A

A bone that becomes a ghost (poor definition of margins) on Tl imaging, but then re-appears
(more morphologically distinct) on T2, or after giving IV contrast, is more likely to have
osteomyelitis.

405
Q

Discitis/Osteomyelitis

overview

A
Infection of the disc and
infection of the vertebral
body nearly always go
together.
The reason has to do
with the route of seeding
406
Q

Discitis/Osteomyelitis

mechanism

A
(1) Seeding of the
vertebral endplate
(which is vascular)
(2) Eruption and
crossing into the
disc space
(3) Eventual
involvement of the
adjacent vertebral body
407
Q

Discitis/Osteomyelitis

early look

A
  • Plain Film: Hard to see

* MR1: Paraspinal and Epidural inflammation, T2 bright disc signal, and disc enhancement.

408
Q

Discitis/Osteomyelitis

laterl plain film

A
  • Adjacent irregular endplate destruction.

* Disc Space Narrowing

409
Q

Discitis/Osteomyelitis

later mr

A
  • T1 - Dark Marrow
  • T2 - Bright Marrow
  • Post Contrast Enhancement
410
Q

Discitis/Osteomyelitis

trivia

A
  • Adults: the source is usually from a recent surgery, procedure, or systemic infection.
  • Children (younger than 5) it’s usually from hematogenous spread.
  • Step 1 trivia: Staph A is the most common bug, and always think about an IV drug user.
  • Almost always (80% of the time) the ESR and CRP are elevated.
  • Gallium is superior to WBC scan in the spine.
411
Q

Discitis/Osteomyelitis

epidural abscess

A

This is an infected collection between the dura and periosteum.

Classic Appearance:
. T1 Dark, T2 Bright,
• Peripheral Enhancement, &
• Restricted
Diffusion.
Classic Scenario:
• HIV patient
• Bad Diabetic.
412
Q

discitis/osteomyelitis

paraspinal vs epidural abscess

A

paraspinal goes to teh front

epidural goes to the spinal canal

413
Q

Pediatric Discitis / Osteomyelitis

A

Children have direct blood supply to the intervertebral disc, so they can get isolated discitis.
Isolated Discitis is basically never seen in adults. The classic scenario is: kid (younger than 4)
with a upper respiratory infection, now with back pain — usually lumbar.

414
Q

TB

overview

A

This is a special topic (high yield) with regard to MSK infection. It’s not that common,
with < 5% o f patients with TB having MSK involvement. Although on multiple choice
tests, I think yo u ’ll find it appears with a high frequency.

415
Q

TB

potts disease overview

A

(tuberculosis o f the spine)
The vertebral body is involved with sparing o f
the disc space until late in the disease (very
different than more common bacterial infections).

416
Q

TB potts disease things to know

A

It tends to spare the disc space
It tends to have multi-level thoracic “skip ” involvement
Buzzword “Large paraspinal abscess
Buzzword “Calcified Psoas Abscess ”
Buzzword “Gibbus Deformity ” - which is a destructive fo c a l kyphosis

417
Q

TB

gibbus deformity

A

This is a focal
kyphosis seen in “Pott
Disease” , among
many other thing

418
Q

TB

Potts dz mimic

A

Brucellosis (unpasteurized milk
from an Amish P e rso n ), can also have disc
space preservation.

419
Q

“ Tuberculosis Dactylitis ” (Spina Ventosa)

A

Typically seen in kids with involvement o f the short tubular bones o f the hands and feet.

It is often a smoldering infection without periosteal
reaction.

Classic look is a diaphyseal expansile lesion with soft tissue swelling.

420
Q

S e p tic A rth ritis

overview

A

You see this the most in large joints which have an abundant blood supply to the metaphysis
(shoulder, hip, knee).
IV drug users will get it in the SI joint, an d sternoclavicular joint.

421
Q

S e p tic A rth ritis

risk factors

A

Conventional risk factors include being old, having AIDS, RA, and prosthetic joints.

422
Q

S e p tic A rth ritis

plain film

A

On plain film you might see a joint effusion, or
MRI will show synovial enhancement. If
untreated this will jack your joint in less than
48 hours.

423
Q

N e c ro tiz in g F a s c iitis

A

This is a very bad actor that kills very quickly. The good news is that it’s pretty rare,
typically only seen in HI Vers, Transplant patients, diabetics, and alcoholics. It’s usually polymicrobial (the second form is Group A Strep).
Gas is only seen in a minority o f cases, but if you see gas in soft tissue this is what they want. Diffuse fascial enhancement is what yo u ’d see if the ER is dumb enough to order cross sectional imaging (they often are).

Fournier Gangrene is what they call it in the scrotum ( “testes satchel” ).

424
Q

Pneumoarthrogram Sign

A

If you can demonstrate air within a joint
you can exclude a joint effusion.
No joint effusion
No septic joint

425
Q

Aggressive bone lesions

Helms

A

According to Helms, the wide zone of transition is the
best sign that a lesion is aggressive. This is actually a useful pearl. The simplest way to conceptualize this is
to ask yourself if you can trace the edges of the lesion with a pencil. If you can the lesion is probably benign. If the edges are blurry or there is a gradient to the edge - this is a more likely an aggressive lesions.

426
Q

Aggressive bone lesions

MArgins

A

The reason is that margins reflect the growth the lesion. Bones are dumb. They really only know how to do two things - make bone and destroy bone. The margins are the reflection of bone formation. If the margins are sharp and sclerotic, this means the bone has time to adjust to the irritation and lay down a coat of mature bone. If the margins are not distinct (zone of transition is wide) this indicates a faster growing lesion and therefore a higher probability of malignancy (or infection).

427
Q

Aggressive bone lesions

growth

A

If the tumor grows rapidly enough it can break through the cortex and destroy thenewly formed bone capsule / lamellated bone. When this happens you end up with a triangular structure - called the Codman triangle (as shown in diagram).

428
Q

Aggressive bone lesions

Lytic

A

When we think about bone lesions, we often imagine lytic holes or bone destruction. Bone destruction occurs from complex methods best understood as either direct obliteration via the tumor or pissed off osteoclasts enraged by the uninvited tumor / hyperemia. Trabecular bone loss occurs more rapidly (relative to cortical bone), but is noticed later because cortical bone is more smooth and organized. Supposedly you need to destroy 70% of the trabecular bone before it is noticed radiographically.

429
Q

Aggressive bone lesions

bone destruction

A

Bone destruction that occurs in a uniform geographic pattern (especially with a sharp well defined border) is more suggestive of a benign slow growing lesion. A moth-eaten (cluster of small lytic holes) or permeative (ill-defined tiny oval or streak like (luccncies) suggests rapid infiltrative tumor growth — as seen in myeloma, lymphoma, and Ewings sarcoma. It is worth noting that osteomyelitis and hyperparathyroidism can also demonstrate Moth Permeative these aggressive patterns - pre-test probably is always important.

430
Q

Osteosarcoma

overview

A

There are a bunch o f subtypes, but for the purpose o f this discussion there are 4. Conventional Intramedullary (85%), Parosteal (4%), Periosteal (1%), Telangiectatic (rare). All the subtypes produce bone or osteoid from neoplastic cells. Most are idiopathic but you can have secondary causes (usually seen in elderly) XRT, Pagets, Infarcts, etc …

431
Q

Osteosarcoma

conventional intramedullary

A

More common, and higher grade than the surface subtypes (periosteal, and parosteal). Primary subtypes typically occur in young patients
(10-20). The most common location is the femur (40%), and proximal tibia (15%).

432
Q

Osteosarcoma

buzzwords (conventional)

A

“S u n b u rst”- periosteal reaction that is aggressive and looks like a sunburst

Codman triangle - With aggressive lesions, the periosteum does not have time to ossify completely with new bone (e.g. as seen in single layer and multi-layered periosteal reaction), so only the edge o f the raised periosteum will ossify - creating the appearance o f a triangle.

Lamellated (onion skin reaction) - multi layers o f parallel periosteum, looks like an onion’s skin.

433
Q

Osteosarcoma

trivia

A

Osteosarcoma met to the lung is a “classic” cause o f occult pneumothorax.

434
Q

Osteosarcoma

reverse zoning phenomenon

A

more dense mature matrix in the center, less peripherally (opposite o f myositis ossificans)

435
Q

Osteosarcoma

reverse zoning phenomenon

A

more dense mature matrix in the center, less peripherally (opposite o f myositis ossificans)

436
Q

P a ro s te a l O s te o s a rc om a

A

Generally low grade, BULKY parosteal bone formation.
Think B ig … ju st say Big. This guy loves the posterior distal femur (because o f this location
it can mimic a cortical d esmoid “tug lesion ” early on). The lesion is metaphyseal 90% o f the
time. The buzzword is “string sign” - which refers to a radiolucent line separating the bulky
tumor from the cortex.

437
Q

P e rio s te a l

O s te o s a rc om a :

A
Worse prognosis than parosteal
but better than conventional
osteosarcoma. Tends to occur
in the diaphyseal regions,
classic medial distal femur.
438
Q

Parosteal this vs that

A

early adult/middle age

metaphyseal

likes posterior distal femur

marrow extension

low grad

439
Q

Periosteal this vs that

A

15-25

diaphyseal

likes medial distal femur

usually no marrow extension

intermediate grade

440
Q

T e la n g ie c ta tic O s te o s a rc om a :

A

About 15% have a narrow zone o f transition. Fluid-Fluid levels on MRI is classic. They are High on T1 (from methemoglobin). Can be
differentiated from ABC or GCT (maybe) by tumor nodularity and enhancement.

441
Q

Bone lesion fluid-fluid level DDX

A

Telangiectatic
Osteosarcoma
Aneurysmal Bone Cyst
Giant Cell Tumor

442
Q

Chondrosarcoma

overview

A

Usually seen in older adults (M>F). Likes flat bones,
limb girdles, proximal tubular bones. Can be central
(intramedullary) or peripheral (at the end o f an
osteochondroma). Most are low grade.

443
Q

Chondrosarcoma

risk factors

A

Pagets, and anything cartilaginous

osteochondromas, Maffucci etc…

444
Q

Chondrosarcoma

gamesmanship 1

A

If you want to say chondroblastoma but

it’s an adult think clear cell chondrosarcoma

445
Q

Chondrosarcoma

gamesmanship 2

A

If shown with CT - they have to
show you some “chondroid
matrix” - “arcs and rings”

446
Q

THIS vs THAT
Enchondroma vs Low Grade
Chondrosarcoma

A

Factors Favoring
Chondrosarcoma:

Pain
Cortical Destruction
Scalloping of > 2/3 of the cortex
>5cm in Size
“Changing Matrix”
447
Q

Ewings

overview

A

Extremely rare in African-Americans. Likes to met bone to bone (skip lesions are more common
in Ewings, relative to Osteosarcoma). Does NOT form osteoid from tumor cells, but can mimic
osteosarcoma because of its marked sclerosis (sclerosis occurs in the bone only, not in the soft
tissue - which is NOT the case in osteosarcoma).

448
Q

Ewings

imaging

A

Permeative lesion in the diaphvsis of a child = Ewings

could also be infection, or EG

449
Q

Chordoma

overview

A

Usually seen in adults (30-60), usually slightly younger in the clivus and slightly older in the
sacrum. Most likely questions regarding the chordoma include location (most common sacrum,
second most common clivus, third most common vertebral body), and the fact that they are verv
T2 bright.

450
Q

Chordoma

most commons

A
  • Most common primary malignancy of the spine.
  • Most common primary malignancy of the sacrum.
  • When involving the spine, most common at C2.
451
Q

I say chordoma you say

A

Midline, Midline, Midline!’’
Why is it always midline? It’s made of cells left over from the “notochord” (some embryology
bullshit that was involved in making MIDLINE structures).
Chordomas arc NEVER EVER seen off the midline (NEVER in the hip, leg, arm, hand, etc…).

452
Q

Lucent lesions

overview

A

FEGNOMASHIC is a “useful” mnemonic for lucent bone lesions made popular by Clyde Helms. As
it turns out, you can rearrange the letters of FEGNOMASHIC to form a word FOGMACHINES. I
find it a lot easier to remember a mnemonic if it actually forms a real word. Having said that, the
whole idea of memorizing a list o f 11 or 12 barely related things is stupid. You would never give a
differential that included all o f those, as they occur in different places, in different ages, and often
look very different.
Differentials (for people who know what they are looking at) are usually never deeper than 3 or 4
things. If you are giving a differential o f 12 things, just say you don’t know what it is. Seriously in
the real world bone lesions only come in 3 flavors: 1 - Bad (cancer or infection), 2 - Obviously
benign (not sure I’ll waste saliva mentioning it in my rep o rt), 3 - Ehh hard to tell - get a follow up.
This is actually pretty much true of lesions everywhere in the body. Realizing this allows for the
following paradigm shift:

453
Q

Lucent lesions

multiple choice

A

For multiple choice, when you encounter a lucent bone lesion you can expect one o f two questions (1) what is it ? or (2) what is it associated with ? In either case you are going to need to figure out what it is. In the real world you would probably have to give a short differential, but in the world of multiple choice you will have to come up with one answer. Don’t fret, they have to give you clues so you can pick just one. A useful mental exercise when eliminating multiple choice distractors is to ask yourself “why is it NOT this?” It’s an exercise that is often not performed at the workstation - but very valuable in the test environment - especially for these types o f questions.

454
Q

Lucent lesions

age - key facts

A

. <30 = EG, ABC, NOF,
Chondroblastoma, and Solitary Bone Cysts
• Any Age = Infection
• > 40 = Mets and Myeloma (unless it’s neuroblastoma mets).

455
Q

Lucent lesions

method

A

(1) Age o f the patient? - If you are lucky they will tell you. If you are less lucky you will have to guess. Growth plates open = kid. Growth plates closed with no degenerative change = young adult. Growth plates closed with degenerative changes = older than 40.
(2) Location o f the lesion ? Metaphysis, Epiphysis, Diaphysis? Is this an epiphyseal equivalent discussion (see the next page for discussion).
(3) Classic Locations and Looks ?

456
Q

Lucent lesions

epiphysis

A

In general, only a few lesions tend to arise in the epiphysis. The “four horseman o f the
(e)apophysis” is the mnemonic 1 like to use, and I think about the company AIG that was
involved in some scandal a few years ago. AIG “the evil” Company.

ABC, Infection, Giant Cell, and Chondroblastoma.
* The caveat is that ABC is usually metaphyseal hut after the
growth plate closes it can extend into the epiphysis.

457
Q

lucent lesions

epiphysis malignant

A

For the purpose o f multiple choice tests, it is important to not
forget about the malignant tumor at the end o f the bone
(epiphysis) - Clear Cell Chondrosarcoma. This guy is slow
growing, with a variable appearance (lytic, calcified, lobulated,
ill defined, e tc …). Just remember if they say malignant
epiphyseal you say Clear Cell Chondrosarcoma.

458
Q

Lucent lesions epipyseal equivalents

A

(bones that will have the same lesions as the epiphysis)

Carpals,
Patella,
Greater Trochanter,
Calcaneus

459
Q

Lucent lesions

metaphysis

A

The metaphysis is the fastest growing area o f a bone, with the best blood supply. This
excellent blood supply results in an increased predilection for Mets and Infection. Most o f the
cystic bone lesions can occur in the metaphysis.

460
Q

lucent lesions

diaphysis

A

Just like the metaphysis, most entities can occur in the diaphysis (they ju st do it less).

461
Q

mccune albright

A

polyostotic fibrous dysplasia

girl

cafe au lait spots

precocious puberty

462
Q

mazabraud

A

polyostotic fibrous dysplasia

woman

soft tissue myxomas

increase risk osseous malignant transformation

463
Q

fibrous dysplasia

age

A

The disorder can occur at any age - but the multiple lesion variety “polyostotic” - tends to occur earlier.

You could think monostotic (20’s & 30’s) or polyostotic (< 10 year old). When you see the polyostotic form (often with a mangled horrible horrible face… a face that only a mother could love) - you should think syndromes.

464
Q

adamantinoma

A

A total zebra (probably a unicorn). A tibial lesion that resembles Fibrous dysplasia
(mixed lytic and sclerotic). It is potentially malignant.

465
Q

Nonossifying fibroma

A

These are very common. They arc seen in children, and will spontaneously regress (becoming more
sclerotic before disappearing). They are rare in children not yet walking. Just like GCTs they like to
occur around the knee. They are classically described as eccentric with a thin sclerotic border
(remember GCTs don’t have a sclerotic border). They are called fibrous cortical defects when
smaller than 2 cm.

466
Q

nonossifying fibroma

vocab

A

NOFs are the larger version (> 3cm) of a fibrous cortical defect (FCD). A wastebasket term
for the both of them is simply “fibroxanthoma.”

467
Q

a ffe -C am p a n a c c i Syndrome

A

Syndrome of multiple NOFs, cafe-au-lait

S? sf spots, mental retardation, hypogonadism, and cardiac malformations.

468
Q

Enchondroma

overview

A

This guy is a tumor of the medullary cavity composed of hyaline cartilage. They become progressively more common with age - peaking around 10-30 years old.

The sneaky thing about this lesion is that it looks different depending on the body part it is in.

  • Humerus or Femur = Arcs and Rings
  • Fingers or Toes = Lytic

The enchondroma is actually the most common cystic lesion in the hands and feet. Just like fibrous dysplasia, this lesion does not have periostitis.

469
Q

Enchondroma

appearance

A

The ARCS AND RINGS is the more classic textbook look with the irregularly
speckled calcification of chondroid matrix. Just don’t forget that this classic matrix
is not found in the fingers or toes.

470
Q

Differentiating Enchondroma vs Low Grade Chondrosarcoma

Strategies to Deal with the Chondroid Matrix Lesion in a Long Bone

primary tactic

A

History of pain — Enchondroma vs a Low Grade Chondrosarcoma

Not Painful PAINful (respectively)

471
Q

Differentiating Enchondroma vs Low Grade Chondrosarcoma
▼ f Strategies to Deal with the Chondroid Matrix Lesion in a Long Bone

secondary tactic

A

Enchondroma vs a Low Grade Chondrosarcoma

1-2 cm > 4-5 cm (respectively)

472
Q

Differentiating Enchondroma vs Low Grade Chondrosarcoma
▼ f Strategies to Deal with the Chondroid Matrix Lesion in a Long Bone

tertiary tactic

A

Glitch in the matrix - Enchondroma vs a Low Grade Chondrosarcoma

Arcs & Rings
Arcs & Rings Pattern

Pattern does NO T Changes -
Change around grows etc…

473
Q

Differentiating Enchondroma vs Low Grade Chondrosarcoma
▼ f Strategies to Deal with the Chondroid Matrix Lesion in a Long Bone

tertiary tactic

A

Glitch in the matrix - Enchondroma vs a Low Grade Chondrosarcoma

Arcs & Rings
Arcs & Rings Pattern

Pattern does NO T Changes -
Change around grows etc…

474
Q

Ollier disease

A

multiple enchondromas (3 or more)

slight increase in risk in chondrosarcoma

475
Q

Maffucci syndrome

A

“Marffucci Has More”
More Cancer Risk and More Vascular Malformations

multiple enchondromas

Hemangiomas
(bunch lucent centered calcifications)

Increase risk in Chondrosarcoma
(probably more than Oilier)

476
Q

Eos in oph ilic G ran u lom a (EG):

overview

A

This is typically included in every differential for people less than 30 (peak age is 5-10).
It can be solitary (usually) or multiple.

477
Q

Eos in oph ilic G ran u lom a (EG):

classic appearances

A

(1) Vertebra plana in a kid
(2) Skull with lucent “beveled edge” lesions (also in a kid).
(3) “Floating Tooth” with lytic lesion in alveolar ridge — this would be a differential case

The appearance is highly variable and can be lytic or blastic, with or without a sclerotic border,
and with or without a periosteal response. Can even have an osseous sequestrum.

478
Q

lucent lesions

Classic DDx for Vertebra Plana

A

mets/myeloma

eg

lymphoma

trauma/TB

(MELT)

479
Q

lucent lesions

Classic DDx for Osseous Sequestrum:

A

osteomyelitis

lymphoma

fibrosarcoma

EG

^osteoid osteoma can mimic a sequestrum

480
Q

Giant cell tumors

key criteria

A
  • Physis MUST be closed
  • Non Sclerotic Border
  • Abuts the articular surface
481
Q

Giant cell tumors

trick

A

Another trick is to show you a pulmonary met, and ask if it could be a GCT? The answer is
yes (although this is rare) GCT is considered “quasi-malignanf ’ because it can be locally
invasive and about 5% will have pulmonary mets (which are still curable by resection). As a
result o f this, it should be resected with wide margins.

482
Q

Giant cell tumors

things to know

A
  • Most common in the knee - abutting the articular surface
  • Most common at age 20-30 * physis must be closed
  • There is an association with ABCs (they can turn into them)
  • They are “quasi-malignant” - 5% have lung mets
  • Fluid levels on MRI
483
Q

Giant cell tumors

things to know

A
  • Most common in the knee - abutting the articular surface
  • Most common at age 20-30 * physis must be closed
  • There is an association with ABCs (they can turn into them)
  • They are “quasi-malignant” - 5% have lung mets
  • Fluid levels on MRI
484
Q

Osteoid osteoma

classic

A

pain at night relieved by aspirin

485
Q

Osteoid osteoma

classic age

A

Adolescent” — 10-25 ish.

486
Q

Osteoid osteoma

classic look

A

Oval lytic lesion (“ lucent nidus”) surrounded by dense sclerotic cortical bone (“periosteal
reaction”).

487
Q

Osteoid osteoma

classic location

A

(1) Meta/diaphysis o f long bones (femoral neck = most common) and (2) Posterior elements of the spine (lumbar > cervical > thoracic). Technically the
fingers arc more common than the spine, but that’s rarely show on multiple choice.

488
Q

Osteoid osteoma

mri

A

“Lots o f edema.” I’ll say that again “large amount o f edema fo r the size o f the lesion. ”

Adjacent soft tissue edema is also common - don’t let that fool you.

489
Q

Osteoid osteoma

nuke bone scan

A

Double Density Sign” - very intense central activity at nidus, surrounded
by less intensity o f reactive bone. A common distractor is a stress fracture. Stress fractures
are linear. O.O. should be round.

490
Q

Osteoid osteoma

scoliosis trivia

A

When you have them in the spine (most common in the posterior elements o f the lumbar spine), you frequently have an
associated painful scoliosis with the convexity pointed away from
the lesion.

491
Q

Osteoid osteoma

treamtent

A

These can be treated with percutaneous radiofrequency ablation (as long as it’s not within 1 cm
o f a nerve or other vital structure - typically avoided in hands, spine, and pregnant patients).

492
Q

Osteoid osteoma

associations

A

Painful Scoliosis

Growth Deformity: Increased length and girth of long bones

Synovitis: Can be seen if intra-articular, joint effusions

Arthritis: Can occur from
primary synovitis, or secondarily from altered
joint mechanics.

493
Q

O s te o b la s tom a

A

Basically it’s an osteoid osteoma that is larger than 2 cm. It’s seen in patients < 30 years old.
They are most likely to show this in the posterior elements. It also occurs in the long bones
(35%) and when it does it is usually diaphyseal (75%).

494
Q

A n eu ry sm a l Bone Cyst (ABC):

overview

A

Aneurysmal bone cysts are aneurysmal lesions o f bone with thinwalled,
blood-filled spaces (fluid-fluid level on MR1). Patients are usually < 30. They may develop following trauma.

They can be described as primary ABC, presumably arising denovo or secondary ABC, associated with another tumor (classic GCT). They are commonly associated with other benign lesions.

495
Q

A n eu ry sm a l Bone Cyst (ABC):

thinks to knowq

A
  • Up to 40% o f secondary ABC’s are associated with giant cell tumor o f bone.
  • It’s on the DDx for Fluid - Fluid Level on MRI
  • Patient < 30
  • Tibia is the most common site
496
Q

A n eu ry sm a l Bone Cyst (ABC):

location

A

Location: Tibia > Vert > Femur > Humerus

497
Q

Classic DDx for
Lucent Lesion in
Posterior Elements:

A

Osteoblastoma
ABC
TB

498
Q

S o lita ry (U n ic am e ra l) Bone Cyst

overview

A

It would be unusual to see one o f these in a patient older than 30. Most common in the tubular bones (90-95 %) usually humerus or femur. Unique feature: “Always located centrally.”

499
Q

S o lita ry (U n ic am e ra l) Bone Cyst

imaging

A

It’s going to be shown one o f two ways: (1) With a fracture through it in the humerus (probably with a fallen fragment sign) or (2) As a lucent lesion in the calcaneus (probably with a fallen fragment sign).

500
Q

S o lita ry (U n ic am e ra l) Bone Cyst

fallen fragment sign

A

(bone fragment in the dependent portion o f a lucent bone lesion) is
pathognomonic o f solitary bone cyst.

501
Q

Brown T um o r (H y p e rp a ra th y ro id ism ):

ovvervview

A

The “brown tumor” represents localized accumulations o f giant cells and fibrous tissue (in
case someone asks). They appear as lytic or sclerotic lesions with other findings o f hyperparathyroidism (subperiosteal bone resorption). In other words, they need to tell you he/
she has hyperparathyroidism first. They may ju st straight up tell you, or they will show you some bone resorption first (classically on the side o f a finger, edge o f a clavicle, or under a
rib).

502
Q

Brown T um o r (H y p e rp a ra th y ro id ism ):

healing

A

These things have different stages o f healing / sclerosis. They resorb and can become totally sclerotic / healed, when the Hyper PTH is treated.

503
Q

Chondroblastoma

overview

A

This is seen in kids (90% age 5-25). They classically show it in two ways (1) in the epiphysis o f the tibia on a 15 year old, or (2) in an epiphyseal equivalent.

504
Q

what are epiphyseal equivalents

A
  • Patella
  • Calcaneus
  • Carpal Bones
  • A n d all the Apophyses (greater an d less trochanter, tuberosities, etc…)
505
Q

Chondroblastoma

features

A

A thin sclerotic rim, extension across the physeal plate
(25-50%), periostitis (30%). Actual location: femur > humerus > tibia . This may show bone
marrow edema, and soft tissue edema on MR1 (MRI can mislead you into thinking it’s a bad
thing). This is one o f the only bone lesions that is often NOT T2 bright. They tend to
reoccur after resection (like 30% o f the time).

506
Q

Chondroblastoma

gamesmanship hip

A

When you have a chondroblastoma in the hip, it tends fa v o r the greater trochanter (more than the femoral epiphysis).

507
Q

Chondroblastoma

gamesmanship hip

A

When you have a chondroblastoma in the hip, it tends fa v o r the greater trochanter (more than the femoral epiphysis).

508
Q

C hondromyxoid Fibroma

A

This is the least common benign lesion o f cartilage. It is usually in patients younger than 30.
The typical appearance is an osteolytic, elongated in shape, eccentrically located,
metaphyseal lesion, with cortical expansion and a “bite” like configuration. Sorta looks like
an NOF - with the classic location in the proximal metaphyseal region o f the tibia.

509
Q

Lucent lesions

greater trochanter - hip

A

Remember this is an epiphyseal equivalent and the chrondroblastomas prefer it to the femoral epiphysis. You can get all the other DDxs (ABC, Infection, GCT, etc … here as well). Plus, you can have avulsions o f the gluteus medius and minimus.

510
Q

lucent lesions lesser trochanter - hip

A

An avulsion here without significant clinical history should make you think pathologic fracture.

511
Q

Lucent lesions

intertrochanteric region - hip

A

Classic DDx here: Lipoma, Solitary Bone Cyst, and Monostotic Fibrous Dysplasia.

512
Q

Classic (& sneaky non-classic) Lesions that can be shown in the calcaneus

geod

A

Older Patient + Subtalar degenerative change / Obvious Arthritis

513
Q

Classic (& sneaky non-classic) Lesions that can be shown in the calcaneus

osteomyelities/mets

A

The calcaneal apophysis (equivalent to the metaphyseal region o f long bones) will have a similar predilection for collection hematogenous spread of both infection or cancer (GU or Colon).

In both cases the involvement favors the posterior meta-epiphvseal region (which has the richest blood supply), with lesions potentially growing large enough to involve the entire calcaneus.

514
Q

Classic (& sneaky non-classic) Lesions that can be shown in the calcaneus

overview

A

The suggested Promethean method is to first use
location within the bone, and then use characteristic
appearance as a secondary discriminator.
First let us take a closer look at the calcaneus.
Remember this thing is an epiphyseal equivalent, but
only in certain locations. It’s probably better to think
about the bone like a hybrid long bone - complete
with a diaphysis, two metaphysis, and three epiphysis

515
Q

Classic (& sneaky non-classic) Lesions that can be shown in the calcaneus

chondroblastoma

A

Most classic epiphysis lesion - with a preference for the superior epiphysis near the talocalcaneal articulation (although they can be at any of the 3 epiphysis).

Lucent lesion, that can have some internal calcifications.

516
Q

Classic (& sneaky non-classic) Lesions that can be shown in the calcaneus

giant cell tumor

A

Can also involve the epiphysis (although it typically starts out metaphyseal and grows into the epiphysis). Remember these things required a closed physis. The Posterior Metaphysis / Epiphysis is favored.

517
Q

Classic (& sneaky non-classic) Lesions that can be shown in the calcaneus

osteoid osteoma

A

Talus > Calcaneus

Similar to Chondroblastoma in favoring the superior epiphysis near the talocalcaneal articulation.

Distinction is the sclerotic thickening of the adjacent bone and the radiolucent nidus.

518
Q

Classic (& sneaky non-classic) Lesions that can be shown in the calcaneus

pseudo-cyst

A

This is a variation on the normal trabecular pattern, which creates a central triangular radiolucent area. This area is sometimes called the “pseudo-cyst triangle” and is obnoxiously located in the same anterior 1/3 as the SBC and Lipoma. Supposedly the persistence of thin trabeculae, visible nutrient foramen, and the classic location are helpful in telling it from the other benign entities.

519
Q

bone lesions

mets overview

A

Should be on the differential for any patient over 40 with a lyticlesion. As a piece o f trivia renal cancer is ALWAYS lytic (usually).

520
Q

bone lesions classic blastic lesions

A

prostate, carcinoid, medulloblastoma

521
Q

bone lesions classic lytic lesions

A

renal and thyroid

522
Q

bone lesions next step

A

Prostate Met vs Bone Island??? - Get a Bone Scan
• Bone Island should be mild (or not active)
• Prostate Met should be HOT

523
Q

M u ltip le M y e lom a (MM )

overview

A

Plasma cell proliferation increases surrounding osteolytic
activity (in case someone asks you the mechanism). Usually in older patient (40’s-80’s).
Plasmacytomas can precede clinical or hematologic evidence o f myeloma by 3 years.

524
Q

M u ltip le M y e lom a (MM )

imaging

A

They usually have discrete margins, and can be solitary or multiple. Vertebral body
destruction with sparing o f the posterior elements is classic. Bone Scan is often negative,
skeletal survey is better (but horribly painful to read), and MRI is the most sensitive.

525
Q

M u ltip le M y e lom a (MM )

classic scenario

A

MM manifesting as Diffuse Osteopenia

526
Q

myeloma related conditions

plasmacytoma

A

(usually under 40): This is a discrete, solitary mass o f neoplastic monoclonal
plasma cells in either bone or soft tissue (extramedullary subtype). It is associated with latent
systemic disease in the majority o f affected patients. It can be considered as a singular
counterpart multiple myeloma. The lesions look like a geographic lytic area, sometimes with
expansile remodeling.

527
Q

mini brain appearance

A

Plasmacytoma in vertebral body

528
Q

POEMS

A

This is basically “Myeloma with Sclerotic Mets.” It’s a rare medical syndrome with plasma cell proliferation (typically m y e lom a ), neuropathy, and organomegaly.

529
Q

POEMS

A

This is basically “Myeloma with Sclerotic Mets.” It’s a rare medical syndrome with plasma cell proliferation (typically m y e lom a ), neuropathy, and organomegaly.

530
Q

Lucent Lesion Classic Looks and Locations

Long Lesion in a Long Bone

A

Fibrous dysplasia

531
Q

Lucent Lesion Classic Looks and Locations

ground glass

A

fibrous dysplasia

532
Q

Lucent Lesion Classic Looks and Locations

lyst lesion with a hazy matrix

A

fibrous dysplasia

533
Q

Lucent Lesion Classic Looks and Locations

chondroid matrix in the proximal humerus or distal femur

A

cnchondroma

534
Q

Lucent Lesion Classic Looks and Locations

lucent lesion int he finger or toe

A

enchondroma

535
Q

Lucent Lesion Classic Looks and Locations

epiphyseal tibial lesion in a teenager

A

chondroblastoma

536
Q

Lucent Lesion Classic Looks and Locations

epiphyseal equivalent lesion

A

chondroblastoma or iant cell tumor

technically GCTs grow into the epiphysis

537
Q

Lucent Lesion Classic Looks and Locations

lucent lesion in the greater trochanter

A

chondroblastoma

538
Q

Lucent Lesion Classic Looks and Locations

lusent lesion with a gracture (fallen fragment) in the humerus

A

solitary bone cyst

539
Q

Lucent Lesion Classic Looks and Locations

calcaneal lesion with central calcification

A

lipoma

540
Q

Lucent Lesion Classic Looks and Locations

lucent lesion in the skull

A

eg

541
Q

Lucent Lesion Classic Looks and Locations

vertebra plana in a kid

A

eg

542
Q

Lucent Lesion Classic Looks and Locations

vertebra plana in an adult

A

mets

543
Q

Lucent Lesion Classic Looks and Locations

sequestrum/nidus in the tibia/femur

A

osteoid osteom

544
Q

Lucent Lesion Classic Looks and Locations

painful scoliosis

A

osteoid osteoma

545
Q

Lucent Lesion Classic Looks and Locations

calcificed lesion in the posterior element of the c spine

A

osteoblastoma

546
Q

Solitary vs Multiple (Generalization for Multiple Choice Trivia)

multiple sclerotic lesions

A

mets

547
Q

Solitary vs Multiple (Generalization for Multiple Choice Trivia)

multiple sletoritc lesions centered around a joint

A

osteopoikolosis

548
Q

Solitary vs Multiple (Generalization for Multiple Choice Trivia)

multiple lucent lesions older than 49

A

Mets, Myeloma, Metastatic Non-Hodgkin Lymphoma

549
Q

Size matters

Nidus < 2.0 cm

A

osteoid ostema

550
Q

Size matters

Nidus > 2.0 cm

A

osteoblastoma

551
Q

Size matters

Well-defined lytic lesion in the cortex of a long
bone with a sclerotic rim < 3 cm

A

fibrous corticald efect

552
Q

Size matters

Well-defined lytic lesion in the cortex of a long
bone with a sclerotic rim > 3 cm

A

noonossifying fibroma

553
Q

Size matters

Chondral lesion in a long bone 1-2 cm

A

probably an enchondroma

554
Q

Size matters

Chondral lesion in a long bone > 4-5 cm

A

increased risk of low grade chondrosarcoma

555
Q

Size matters

Chondral lesion in a long bone > 4-5 cm

A

increased risk of low grade chondrosarcoma

556
Q

Liposclerosing Myxofibroma

A

Very characteristic location - at the intertrochanteric region o f the femur.
Looks like a geographic lytic lesion with a sclerotic margin. Despite nonaggressive
appearance. 10% undergo malignant degeneration so they need to
be followed.

557
Q

osteochondroma

overview

A

Some people think o f this as more o f a developmental anomaly (although
they still always make the tumor chapter). Actually, it’s usually listed as the
most common benign tumor (“exostosis”). They can be radiation induced,
making them the only benign skeletal tumor associated with radiation.

558
Q

osteochondroma

malignant transformation

A

They have a very small risk of malignant transformation (which supposedly
can be estimated based on size o f cartilage cap).
Supposedly a cap > 1.5 cm is concerning.

559
Q

osteochondroma

key points

A

Key Points:
• They point away from the joint
• The bone marrow flows freely into the lesion

560
Q

Multiple H e re d ita ry Exostosis

A

AD condition with multiple osteochondromas.

They have an increased risk of malignant transformation.

561
Q

Trevor Disease (Dysplasia Epiphysealis H em ime lic a - DEH):

A

This is a disease characterized by the development of osteochondromas develop at the epiphysis
which result in significant joint deformity (most common in ankle and knee) — making you
terrible at tennis and soccer. Instead o f pointing away from the joint (like a normal
osteochondroma) these assholes point into the joint — this is why you have so many joint
issues. You see this is young children. The osteochondroma looks more like an irregular mass.
They tend to be treated with surgical excision.

562
Q

Supracondylar Spur (Avian Spur):

A

This is an Aunt Minnie, and normal variant. This is anosseous process, that usually does nothing, but can compress the median nerve if the Ligament of Ligament o f Struthers smashes it.

Notice this thing points towards the joint, that is how you know it is not an osteochondroma. Also - it is not a Trevor Disease thing - because

(1) of the characteristic location and
(2) it is not originated from the epiphysis.

563
Q

P e rio s te a l Chondroma (J u x ta -C o r tic a l Chondroma):

A

When you see a lesion in the finger
o f a kid think this. It’s a rare entity, o f cartilaginous
origin. “Saucerization” o f the adjacent cortex with
sclerotic periosteal reaction can be seen.

564
Q

O s teo fib ro u s D ysplasia

A

This is a benign lesion found exclusively in the tibia or
fibula in children (10 and under - usually). It looks like an NOF , but is centered in the
anterior tibia, and has associated anterior tibial bowing. It can occur with Adamantinoma,
and the two cannot be differentiated with imaging

565
Q

When I say looks like NOF in the anterior tibia with anterior bowing

A

yo u say Osteofibrous Dysplasia.

566
Q

Dis ta l Fem ora l M e ta p h y s e a l Ir re g u la r ity (C o r tic a l Desmoid):

overview

A

This is a lucency seen along the back o f the
posteriomedial aspect o f the distal femoral
metaphysis. If they show you a lateral knee xray,
and there is an irregularity or lucency on the
back o f the femur this is it. It’s often bilateral.

567
Q

Dis ta l Fem ora l M e ta p h y s e a l Ir re g u la r ity (C o r tic a l Desmoid):

buzzwords

A

“Scoop like defect” with an

“irregular but intact cortex.”

568
Q

Dis ta l Fem ora l M e ta p h y s e a l Ir re g u la r ity (C o r tic a l Desmoid):

treamtment

A

This is a total incidental finding and is a do n ’t
touch lesion. Don’t biopsy it, Don’t MRI it.
Just leave it alone. If you really want to know,
it’s probably a chronic tug lesion from the
adductor magnus.

569
Q

C a lc ium H y d ro x y a p a tite

overview

A

Most pathologic calcification in the body is calcium hydroxyapatite, which is also the most abundant form o f calcium in bone.

The calcium is deposited in tendons around the joint. The most common location for
hydroxyapatite deposition is the shoulder. Specifically, the supraspinatus tendon is the
most frequent site of calcification, usually at its insertion near the greater tuberosity. The
longus colli muscle (the muscle anterior to atlas -> T3) is also a favorite location fo r multiple
choice test writers. It may be primary (idiopathic) or secondary. Secondary causes worth
knowing are: chronic renal disease, collagen-vascular disease, tumoral calcinosis and
hypervitaminosis D.

570
Q

Calcium hydroxyapatite deposition disease

A

calcific tendinitis.

571
Q

Calcium hydroxyapatite deposition disease

A

calcific tendinitis.

572
Q

O s teo p o ik ilo s is :

A

It’s ju st a bunch o f bone islands. Usually in epiphyses (different from
blastic mets or osteosarcoma mets). It can be inherited or sporadic but if you are forced to
pick a pattern - I’d go with autosomal dominant.

573
Q

O s teo p o ik ilo s is :

trivia

A

Osteopoikilosis tends to be joint centered (clustered around centered). Sclerotic mets will be all over the place. Sclerotic mets believe in nothing Lebowski.

574
Q

mers vs osteopoikilosis

A

Osteopoikilosis patients tend to be keloid formers.

575
Q

E ng elmarm ’s D is e as e

overview

A

This is also known as progressive diaphyseal dysplasia or PDD. What you see is fusiform bony enlargement with sclerosis o f ; 5 j ;
“ the long bones. This is a total zebra that begins in childhood.

576
Q

E ng elmarm ’s D is e as e

things to know

A
  • It’s Bilateral an d Symmetric
  • It likes the long bones - usually shown in the tibia
  • I t s hot on bone scan
  • It can involve the skull — an d can cause optic nerve compression
577
Q

T h a la s s em ia

overview

A

This is a defect in the hemoglobin chain
(can be alpha or beta - major or minor). From the MSK Radiologist prospective, we are talking about “hair-on-end’ skulls, expansion o f the facial bones, “rodent faces,”
expanded ribs “jail-bars” . It is frequently associated with extramedullary hematopoiesis.

578
Q

thalassemia

sinuses

A

will obliterate sinuses

579
Q

sickle cell sinus

A

will not obliterate sinuses

580
Q

thalassemia

lytic

A

Usually Asymptomatic

581
Q

thalassemia

mixed (reparative)

A

Elevated Alkaline Phosphate. Fractures

582
Q

thalassemia

sclerotic (latent inactive)

A

Elevated Hydroxyproline. More fractures. Sarcomas may develop.

583
Q

thalassemia

sclerotic (latent inactive)

A

Elevated Hydroxyproline. More fractures. Sarcomas may develop.

584
Q

AVN of th e Hip

Overview

A

Variety o f causes including Perthes in kids, sickle cell, Gaucher’s, steroid use e tc …. It can also be traumatic with femoral neck fractures (degree o f risk is related to degree o f displacement / disruption o f the retinacular vessels). AVN o f the hip typically involves the superior articular surface, beginning more anteriorly.

585
Q

AVN o f th e Hip:

double line sign

A

Best seen on T2; inner
bright line (granulation
tissue), with outer dark
line (sclerotic bone).

586
Q

AVN o f th e Hip:

rim sign

A

Best seen on T2; high T2 signal line sandwiched between two low signal
lines. This represents flu id between sclerotic borders o f an osteochondral fragment, and implies instability. (Stage III).

587
Q

AVN o f th e Hip:

crescent sign

A

Seen on X-ray (optimally frog leg); Refers to a subchondral lucency seen most frequently in
the anterolateral aspect of the proximal femoral head. It indicates imminent collapse.

588
Q

Plain film stages of osteonecrosis

A
o Zero = Normal
o One = Normal x-ray, edema on MR
o Two = Mixed Lytic / Sclerotic
o Three = Crescent Sign, Articular Collapse, Joint Space Preserved
o Four = Secondary Osteoarthritis
589
Q

P a g e t D is e a s e (O s te itis D e fo rm an s )

overivew

A

A relatively common condition that affects 4% o f people at 40, and 8% at 80 (actually 10%, but easier to remember 8%). M > F. Most people are asymptomatic. The pathophysiology o f Paget is not well understood.

590
Q

P a g e t D is e a s e (O s te itis D e fo rm an s ):

phases

A

The bones go through three phases which progress from lytic to mixed to sclerotic (the latent inactive phase). The phrase “Wide Bones with Thick Trabecula” make you immediately say Pagets (nothing else really does that).

591
Q

P a g e t D is e a s e (O s te itis D e fo rm an s ):

flavors

A

(1) Monostotic and (2) Polyostotic - with the poly subtype beingmuch more common (80-90%).

592
Q

P a g e t D is e a s e (O s te itis D e fo rm an s ):

buzzwords/signs

A

•Blade of Grass Sign: Lucent leading edge in a long bone
*Osteoporosis Circumscripta: Blade of Grass in the Skull
•Picture Frame Vertebra: Cortex is thickened on all sides (Rugger Jersey is only superior and inferior cndplates)
• Cotton Wool Bone: Thick disorganized trabeculae
• Banana Fracture: Insufficiency fracture of a bowed soft bone (femur or tibia).
*Tam O ’Shanter Sign: Thick Skull - with the frontal aspect “falling over the facial bones”
*Saber Shin: Bowing of the tibia
• Ivory Vertebra: This is a differential finding, including mets. Pagets tends to be expansile.

593
Q

P a g e t D is e a s e (O s te itis D e fo rm an s ):

complications

A

Deafness is the most common complication. Spinal stenosis from cortical thickening is very characteristic. Additional complications include cortical stress fracture, cranial nerves paresis, CHF (high output), secondary hyperparathyroidism (10%), Secondary development of osteosarcoma (1%) - which is often highly resistant to treatment. As a piece o f ridiculous trivia - giant cell tumor can arise from Paget.

594
Q

P a g e t D is e a s e (O s te itis D e fo rm an s ):

trivia

A

Of all the tumors to which Paget may devolve

to, Osteosarcoma is the Most Common.

595
Q

P a g e t D is e a s e (O s te itis D e fo rm an s ):

total trivia

A

Paget bone is hypervascular and may be 5
degrees hotter than other bone (get your thermometer
ready). Aik Phos will be elevated (up to 20x) in the
reparative phase.

596
Q

P a g e t D is e a s e (O s te itis D e fo rm an s ):

classic look

A

Expanded Bone

Coarse or Thick Trabecular Pattern

597
Q

P a g e t D is e a s e (O s te itis D e fo rm an s ):

skull

A

Large Areas of Osteolysis in the Frontal and Occipital Bones “Osteoporosis Circumscripta”, in the lytic phase.

The skull will look “cotton wool” in the mixed phase.

Thickened sclerotic appearance is a good chronic look. Involves BOTH inner and outer table (Fibrous Dysplasia favors the outer table)

Tam O ’Shanter Sign: Skull sorta looks like one of those stupid hats with the frontal aspect “falling over the facial bones”

598
Q

P a g e t D is e a s e (O s te itis D e fo rm an s ):

long bones

A

Advancing margin o f lucency from one end to the other is the so-called “blade o f grass” or “flame.” Will often spare the fibula, even in diffuse disease.

Tibia Bowing “saber shin” is also classic.

599
Q

P a g e t D is e a s e (O s te itis D e fo rm an s ):

pelvis

A

Most common bone involved. “Always” involves the iliopectineal line on the pelvic brim.

Can cause advanced arthritis and acetabular protrusio.

classic look on bone scan

600
Q

P a g e t D is e a s e (O s te itis D e fo rm an s ):

spine

A

Cortical Thickening can cause a “picture
frame sign” (same as osteopetrosis). Also
can give you an ivory vertebral body.

601
Q

PAgets

risks

A

The incidence increases with age (around 8% at age 80). It’s at increased risk for fracture, and has a 1% risk o f sarcoma degeneration (usually high grade).

It’s shown two ways in the spine:

(1) An enlarged “ ivory vertebrae”,
(2) Picture frame vertebrae (sclerotic border) — with central lysis (mixed p h a se)

602
Q

Renal Osteodystrophy

overview

A

The
way it’s shown in the spine is the “Rugger Jersey Spine” - with sclerotic bands at the top and
bottom o f the vertebral body. You could also have paraspinal soft tissue calcifications

603
Q

Renal Osteodystrophy

sign

A

rugger jersey, top and botton only

604
Q

Osteopetrosis

A

This is a genetic disease with impaired osteoclastic resorption. You have thick cortical bone, with diminished marrow. On plain film or CT it can look like a Rugger Jersey Spine or Sandwich vertebra. On MR you are going to have loss o f the normal T1 bright marrow signal, so it will be T1 and T2 dark.

605
Q

H-shaped vertebra

A

is usually a buzzword for sickle cell, although it’s only seen in about 10% o f cases. It results from microvascular endplate infarct. If you see “H-Shaped vertebra,” the answer is sickle cell. If sickle cell isn ’t a choice the answer is Gauchers. Another tricky way to ask this is to say which o f the following causes “widening o f the disc space.” Widened disc space is another way o f describing a “ H Shape” without saying that.

606
Q

Pagets

MRI

A

Lytic / Early Mixed:
Heterogenous T2; T1 is isointense to muscle, with a “ speckled appearance”

Late Mixed: Maintained fatty high T1 and T2 signals

Sclerotic: Low signal on T1 and T2

607
Q

Pagets

this vs that malignant vs active disease

A
  • Both are T2 Bright and Will Enhance
  • Malignant Transformation will lose the Normal T1 signal (just like a cancer would)
  • Best Sequence fo r Distinguishing the two? T1 Pre Con
608
Q

Pagets

nuclear medicine

A

The primary utility o f a bone scan is in defining the extent o f disease and to help assess response to treatment. The characteristic look for Paget is “Whole Bone Involvement.”

For example, the entire vertebral body including the posterior elements, or the entire pelvis. The classic teaching is that Paget is hot on all three phases (although often decreased or normal in the sclerotic phase).

609
Q

Tibial Bowing

overview

A

Most likely shown as an Aunt Minnie: NF-1 anterior with a fibular pseudoarthrosis, Rickets with wide growth plates, or Blounts tibia vara. Pagets can also cause this.

The most likely pure trivia question is that physiologic bowing is smooth, lateral, an d occurs from 18 months - 2 years.

610
Q

Tibial bowing

NF1

A

Anterior Lateral - Unilateral

May be unilateral. May have hypoplastic fibula with pseudarthrosis.

611
Q

Tibial bowing

foot deformities

A

Posterior

612
Q

Tibial bowing

physiologic bowing

A

Lateral - Bilateral Symmetric

Self limiting between 18
months and 2 years.

613
Q

Tibial bowing

hypophosphatasia

A

Lateral

Rickets in a newborn”

614
Q

Tibial bowing

rickets

A

Lateral

Fraying o f the metaphyses and widening o f the growth plates.

Seen best in “fast growing bones” - knee, wrist

615
Q

Tibial bowing

blount

A

Tibial Vara - Often asymmetric

616
Q

Tibial bowing

osteogenesis imperfecta

A

Involves all long bones

Early walking, Fat, black kid.

Proximal tibia posteromedial physeal growth disturbance resulting in deformity

617
Q

Tibial bowing

dwarfs

A

Short Limbs

618
Q

SOFT T IS S U E MASSES

overview

A

Here is the simple part - only about 20-30% o f them can be accurately diagnosed on MRI. That’s
because they are almost all T2 bright and enhance. This is actually good news for the purpose o f
multiple choice, because you only need to learn the ones that d o n ’t behave like that - or are
overwhelming likely due to epidemiological stats (which has to be provided for you - either
directly “the patient is 65” or with clues - “arthritis = old” , “no arthritis = not old”).

619
Q

SOFT T IS S U E MASSES

list

A
  • MFH — Malignant Fibrous Histiocytoma aka - Pleomorphic Undifferentiated Sarcoma
  • Synovial Sarcoma
  • Lipoma, Atypical Lipoma, Liposarcoma
  • Hemangioma
  • Myxoma
620
Q

Malignant Fibrous Histiocytoma (MFH)

name

A

Yes - they changed the name to Pleomorphic Undifferentiated Sarcoma “PUS.” However, I want
you to continue to think o f this as MFH because it will help you remember some o f the imaging
features.

621
Q

Malignant Fibrous Histiocytoma (MFH)

overview

A

First, the generalizations — This is very common. It’s seen in old people. It’s seen in a central location (proximal arms and legs).

622
Q

Malignant Fibrous Histiocytoma (MFH)

features

A

About h a lf the time it’s dark to intermediate on T2 (remember most soft tissue tumors are T2 bright). The way 1 remember this is the word “fibrous” - makes me think scar (which is dark).

623
Q

Malignant Fibrous Histiocytoma (MFH)

gamesmanship

A

These things are often associated with spontaneous hemorrhage - they outgrow their blood supply. The history is often “old lady, stood up from a chair” - has a big proximal muscular hematoma — under that hematoma is the MFH.

624
Q

Malignant Fibrous Histiocytoma (MFH)

trivia X2

A

Trivia: Bone infarcts can turn into MFH - ‘’sarcomatous transformation o f infarct”
Trivia: Radiation is a risk factor.

625
Q

Synovial Sarcoma

gamesmanship

A

They occur close to the joint (but not in the
jo in t). To confuse the issue they may have secondary
invasion into the joint (10%), however for the purpose o f
multiple choice tests they “never involve the jo in t.” A
common trick is to show an ultrasound o f the leg with looks
like a Baker’s cyst - but the mother fucker is too complex - or
has flow in it. *Not everything in the popliteal fossa is a
Baker’s cyst - especially on multiple choice.

626
Q

Synovial Sarcoma

ways to show

A

Besides the “not-a-Baker’s cyst” trick - there are 3 other ways to show this. (1) as the “trip le
sign” , which is high, medium, and low signal all in the same mass (probably in the knee) on
T2, (2) as the “ bowl of g ra p e s ” which is a bunch o f fluid -flu id levels in a mass (probably in
the knee), or (3) as a plain x-ray with a soft tissue component and calcifications - this would
be the least likely way to show it.

627
Q

Synovial Sarcoma

trivia

A
  • Most sarcomas d o n ’t attack bones; Synovial Sarcoma Can
  • Most sarcomas present as painless mass; Synovial Sarcomas Hurt
  • Soft tissue calcifications + Bone Erosions are highly suggestive
  • They are slow growing and small in size, often leading to people thinking they are B9.
    90% have a translocation o f X -18.
  • Most common malignancy in teens/young adults o f the foot, ankle, and lower extremity
628
Q

“Ball-like tumor” in the extremity’ o f a yo u n g adult

A

yo u say Synovial Sarcoma

629
Q

Soft Tissue Tumor in the Foot ” o f a yo u n g adult,

A

you say synovial sarcoma

630
Q

Baker’s Cyst

A

Baker’s Cyst MUST be located between the medial head of the gastrocnemius and the semimembranosus.

If it’s NOT - you should think Synovial Sarcoma -and “next step” MRI.

631
Q

Baker’s Cyst

A

Baker’s Cyst MUST be located between the medial head of the gastrocnemius and the semimembranosus.

If it’s NOT - you should think Synovial Sarcoma -and “next step” MRI.

632
Q

Lipoma vs Atypical Lipoma vs Liposarcoma

overview

A

These exist on a spectrum, with Lipoma being totally benign, and Liposarcoma being a bad
bad boy. A pearl is that, histologically, Atypical Lipoma behaves and looks ju st like a low
grade Liposarcoma. It would be total horse-shit to ask you to tell those apart. It’s more likely
the distinction will be either Lipoma vs Liposarcoma.

633
Q

Lipoma vs Atypical Lipoma vs Liposarcoma

pearls

A

•Liposarcomas tend to be DEEP (retroperitoneum)
‘ Liposarcomas tend to be BIG
‘Lipomas tend to be Superficial

634
Q

Lipoma vs Atypical Lipoma vs Liposarcoma

trivia

A

Myxoid Liposarcoma is the most common liposarcoma in patients < 20. They can be
T2 Bright (expected), but T1 dark (confusing) - d o n ‘t call it a cyst. Also, d o n ’t call it a
comeback (I’ve been here for years). They’ll need gad+

635
Q

Lipoma

A

Signal Intensity parallels fat on all sequences.

Will Fat Sat Out

No Sepations (or thin ones)

636
Q

Atypical lipoma/low grade liposarcoma

A

May have parts that are slightly darker (or brighter) than fat on T1.

May incompletely fat sat

Thick Chunky Septations

637
Q

high grade liposarcoma

A

May not even have fat (for the exam it will have some otherwise you can’t even tell for sure that it is a Liposarcoma)

May incompletely fat sat (or not fat sat at all)

Thick Nodular Complex Stuff

Enhancing Components

638
Q

hemangioma

tricks

A

• T2 bright (like most tumors)
• Flow voids. They have to show you flow voids (buncha dark holes).
• Hemangiomas don’t respect fascial boundaries
- they will infiltrate into stuff (this is a somewhat unique feature).
• Enhances Intensely - Duh - they are a vascular tumor
• They can contain fat - and likely will on multiple choice.

639
Q

hemangioma

next step

A

A great next step question would be to ask for a plain film. Why a plain film? phleboliths my friend — If they show you soft tissue phleboliths then hemangioma is the answer.

640
Q

Mazabraud syndrome

A

It’s a totally zebra syndrome - which makes it totally appropriate for an “ intermediate level exam.” It has 3 main findings:
(1) Polyostotic Fibrous Dysplasia - which makes you ugly
(2) Multiple Soft Tissue Myxomas
(3) Difficulty finding a date to the prom
— *see finding “ 1”

641
Q

Myxoma

overview

A

If this shows up on the exam, it is almost certainly going to be shown in the setting o f Mazabraud Syndrome.

642
Q

Myxoma

look like

A

They are T2 bright (like every tumor), but tend to be lower signal than muscle on T1 - which makes them sorta unique.

643
Q

CT v s MRI
f o r L e s io n C h a r a c t e r i z a t i o n :

CT is good for

A

-Occult Bone Destruction
-Matrix and Mineralization — Example,
better look at the lucent nidus of an osteoid osteoma.

644
Q

CT v s MRI
f o r L e s io n C h a r a c t e r i z a t i o n :

mr is good for

A
  • Staging — specifically local extend and tumor spread.

- Follow up - to assess response to therapy.

645
Q

Osteosarcoma treatment

A
Chemo first (to kill micro
m e ts ) , followed by wide excision
646
Q

Chondrosarcoma treatment

A

Usually ju st wide
excision (they are usually low grade, and
main concern is local recurrence).

647
Q

Giant Cell tumor treament

A

Because it extends to
the articular surface usually requires
arthroplasty.

648
Q

Ewings treatment

A

Both Chemo and Radiation,

followed by wide excision.

649
Q

Pigmented V illo n o d u la r Synovitis (PVNS)

overview

A

PVNS is an uncommon benign neoplastic process that may involve the synovium o f the joint diffusely or focally. It can also affect the tendon sheath.

650
Q

Pigmented V illo n o d u la r Synovitis (PVNS)

intra-articular disease

A

Basically, it’s Synovial Proliferation + Hemosiderin Deposition.
The knee is by far the most common joint affected (65-80%). On plain film, features you will
probably see are a joint effusion with or without marginal erosions. Osseous erosions with
preservation o f the joint space and normal mineralization is typical. It is not possible to
distinguish PVNS from synovial chondromatosis (see below) on plain film. MRI will be
obvious with blooming on gradient echo, and this is the most likely way they will show this.
Treatment is with complete synovectomy, although recurrence rate is 20-50%.

651
Q

Pigmented V illo n o d u la r Synovitis (PVNS)

trivia

A

Unusual in kids, but when present is typically polyarticular.

652
Q

Pigmented V illo n o d u la r Synovitis (PVNS)

giant cell tumor of the tendon sheath

A

Typically found in the hand
(palmar tendons). Can cause erosions on the underlying bone. Will be soft tissue density, and be
T1 and T2 dark (contrasted to a glomus tumor which is T1 dark, T2 bright, and will enhance
uniformly).

653
Q

Prima ry Synovial Chondromatosis

A
There are both primary and secondary types;
secondary being the result o f degenerative changes in the joint. The primary type is an
extremely high yield topic. It is a metaplastic / true neoplastic process (not inflammatory) that
results in the fonnation o f multiple cartilaginous nodules in the synovium o f joints, tendon
sheaths, and bursea. These nodules will eventually progress to loose bodies. It usually affects
one joint, and that one joint is usually the knee (70%). It is usually a person in their 40 ’s or 50’s.
Joint bodies (which are usually multiple and uniform in size) may demonstrate the ring and arc
calcification characteristic o f chondroid calcification. Treatment involves removal o f the loose
bodies with or without synovectomy.
654
Q

PVNS quick

A

benign

associated with hemarthrosis

never calcifies

655
Q

Synovial chondromatosis quick

A

benign

not associated with hemarthrosis

may calcify

656
Q

S e conda ry Synovial Chondromatosis:

A

A lower yield topic than the primary type.
This is secondary to degenerative change, and typically seen in an older patient. There will be extensive degenerative changes, and the fragments are usually fewer and larger when compared
to the primary subtype.

657
Q

D ia b e tic M yone cros is:

A

This is basically infarction o f the muscle seen in poorly controlled type 1 diabetics. It almost
always involves the thigh (80%), or calf (20%). MRI will show marked edema with
enhancement and irregular regions o f muscle necrosis. You should NOT biopsy this: it
delays recovery time and has a high complication rate.

658
Q

Lipoma A rb o re s c en s

overview

A

This is a zebra that affects the synovial lining o f the joints and bursa.

It’s seen in late adulthood (50’s-70’s), with
the most common location being the
suprapatellar bursa o f the knee. Although
it can develop in a normal knee, it’s
often associated with OA, Chronic RA,
or prior trauma. It’s usually unilateral.
On MRI it’s going to behave like fat - Tl
and T2 bright with response to fat
saturation.

This could also be shown on ultrasound with a “frond-like hyperechoic mass” and associated
joint effusion.

659
Q

Lipoma A rb o re s c en s

buzzword

A

“frond-like” deposition

o f fatty tissue.

660
Q

Lipoma A rb o re s c en s

trick

A

sneaky trick is to show this on gradient
- and have you pick up the chemical shift
artifact at the fat-fluid interface.

661
Q

Bone Biopsy

special considerations

A

Pelvis: Avoid crossing gluteal muscles (may be needed for reconstruction).
Knee: Avoid the joint space via crossing suprapatellar bursa or other communicating bursae. Avoid crossing the quadriceps tendon unless it is involved.
Shoulder: Avoid the posterior 2/3rd (axillary nerve courses post -> anterior, therefore a posterior resection will denervate the anterior 1/3).

662
Q

“Don’t Touch Lesions”
Characteristically Benign Lesions that look Aggressive but are NOT
- and should NOT be biopsied because o f possibly misleading pathology.

A

Myositis ossificans

avulsion injury

coritcal desmoid

synovial herniation pit

663
Q

Myositis ossificans

A

Circumferential calcifications with a lucent center

Can look scary on MRI if imaged early because o f edema, and avid enhancement

664
Q

Avulsion injury

A

Typical location near the pelvis

Can have an aggressive periosteal reaction

665
Q

Cortical desmoid

A

Characteristic location on the posterior medial epicondyle o f the distal femur.

Bilateral 30% o f the time.

Can be hot on bone scan.

NOT a desmoid (despite the name). It’s actually a tug lesion from the medial gastrocnemius and ADDuctor magnus.

666
Q

Cortical desmoid

A

Characteristic location on the posterior medial epicondyle o f the distal femur.

Bilateral 30% o f the time.

Can be hot on bone scan.

NOT a desmoid (despite the name). It’s actually a tug lesion from the medial gastrocnemius and ADDuctor magnus.

667
Q

Synovial Herniation Pit

“Pitt’s Pit”

A

Characteristic location in the anterosuperior femoral neck.

Lytic appearing lesion

Associated with femoral acetabular syndrome (probably).

668
Q

Arthritis

epidemiology

A

You can broadly categorize arthritis into 3 categories:
Degenerative (OA, Neuropathic)
Inflammatory (RA and Variants)
Metabolic (Gout, CPPD)

669
Q

Osteoarthritis

A

most common cause. The pathogenesis is that you have mechanical breakdown
(hard work) which leads to cartilage degeneration (fissures, micro-fractures) and fragmentation of
subchondral bone (sclerosis and subchondral cysts). You get all the classic stuff, joint space
narrowing (NOT symmetric), subchondral cysts, endplate changes, vacuum phenomenon, etc… The
poster boy is the osteophyte.

670
Q

Neuropathic Joint

A

The way the case is classically shown is a bad joint followed by the reason for a
bad joint (syringomyelia, spinal cord injury, etc…). A way to think about this is “osteoarthritis with a
vengeance A The buzzword is “Surgical Like Margins.” Basically nothing else causes this kind of
destruction. I like to describe the joints as a deformity, with debris, and dislocation, having dense
subchondral bone, and destruction of the articular cortex. The classic scenario is a shoulder that looks
like it’s been amputated, and then they show you a syrinx.

671
Q

Charcot Foot

A

The classic example of a diabetic neuropathic foot with
the deformity, with debris, and dislocation, having dense subchondral
bone, and destruction of the articular cortex - favoring the midfoot
eventually causing a “rocker-bottom deformity” of the foot resulting
from the collapse of the longitudinal arch.

672
Q

This vs That - Charot vs Infection

A

Diabetics get neuropathic feet and infections - so there can be overlap.
To tell them apart you can look for the presence of an ulcer or sinus tract (that infers infection).
Location is helpful - charcot prefers the midfoot (osteomyelitis prefers the pressure points of the forefoot -
metatarsal heads, IP joints - and the posterior plantar aspect of the calcaneus).

673
Q

Erosive Osteoarthritis

A

(Inflammatory
Osteoarthritis). The buzzword is “gull wing”,
which describes the central erosions. It is seen in
postmenopausal women and favors the DIP joints.

674
Q

Rheumatoid Arthritis

overview

A

There is a ton o f trivia
related to this disease. It’s not a disease o f bone production. Instead it is characterized by
osteoporosis, soft tissue swelling, marginal erosions and uniform joint space narrowing. It’s
often bilateral and symmetric. Classically spares the DIP joints (opposite o f erosive OA).

675
Q

Rheumatoid Arthritis

trivia

A

The 5th Metatarsal head is the first spot in the foot

676
Q

Rheumatoid Arthritis

in hte hand

A

Expect the PIP joints to be involved AFTER the MCP joints. The
First CMC is classically spared (or is the last carpal to be involved). The first CMC should
NOT be first. Obviously OA loves the first CMC so this is helpful in separating them.
Psoriasis, on the other hand, also tends to make the first CMC go last.

677
Q

felty syndrome

A

RA > 10 years + Splenomegaly + Neutropenia

678
Q

caplan syndrome

A

RA + Pneumoconiosis

679
Q

Rheumatoid Variants

A
  • Psoriatic Arthritis
  • Reiter’s syndrome (Reactive arthritis)
  • Ankylosing Spondylitis
  • Inflammatory Bowel Disease
680
Q

Psoriatic arthritis

A

This is seen in 30% o f patients with psoriasis. In almost all cases
(90%) the skin findings come first, then you get the arthritis. As a point o f trivia, there is a
strong correlation between involvement o f the nail and involvement o f the DIP joint. The
classic description is “erosive change with bone proliferation (IP joints > MCP joints).
The erosions start in the margins o f the joint and progress to involve the central portions
(can lead to a “pencil sharpening” effect). The hands are the most commonly affected
(second most common is the feet). Up to 40% o f cases will have SI joint involvement
(asymmetric).

681
Q

Psoriatic arthritis

buzzwords

A

“Fuzzy Appearance” to the bone around the joint (bone proliferation)

Sausage Digit - whole digit has soft tissue swelling

Ivory Phalanx - sclerosis and/or bone proliferation (most commonly the great toe)

Pencil in Cup Deformities

Ankylosis in Finger

“Mouse Ears”

Acro-osteolysis

682
Q

RA quick

A

symmetric

proximal (favors MCP, carpals)

osteoporosis

can cause mutilans when severe

683
Q

psoriasis quick

A

asymmetric

distal (favors ip joints)

no osteoporosis

bone proliferation (the form or periostitis)

can cause mutilans when severe

684
Q

M u tila n s

A
Severe bone resorption leading to
soft tissue “telescoping” collapse.
Trivia: If you
pulled on the
patient’s fingers
they would
lengthen - but who
would want to
touch a patient ?
Yuck!
685
Q

Reiter’s

overview

A

Apparently Reiter was a Nazi (killed a bunch o f people with typhus vaccine experiments). So, people try not to give him any credit for things (hence the name change to Reactive arthritis). Regardless o f what you call it, it’s a very similar situation to Psoriatic arthritis - both have bone proliferation, erosions, and asymmetric SI jo in t involvement.

The difference is that Reiter’s is rare in the hands (tends to affect the feet more). Just remember Reiter’s favors things below the waist (like the penis = urethritis, and the foot).

686
Q

Reiter’s

triad

A

Urethritis
Conjunctivitis
Arthritis

(Can’t See or Climb a Tree to Pee on a Nazi named Reiter).

687
Q

Ank y los in g Spo n d y litis

Overview

A

This disease favors the spine and SI joints. The classic buzzword is “bamboo spine” from the syndesmophytes flowing from adjacent vertebral bodies. Shiny comers is a buzzword, for early involvement. As you might imagine, these spines are susceptible to fracture in trauma. SI joint involvement is usually the first site (symmetric). The joint actually widens a little before it narrows. As a point o f trivia, these guys can have an upper lobe predominant interstitial lung disease, with small cystic spaces.

688
Q

Ank y los in g Spo n d y litis

next step

A

Any significant Ank Spon / DISH + Even Minor Trauma = Whole Spine CT

689
Q

Random High Yield Topic: Ankylosing Spondylitis in the Hip

A

When the peripheral skeleton is involved in patient’s with Ank Spond, think about the shoulders and hips (hips more common). Hip involvement can be very disabling.

Heterotopic Ossification tends to occur post hip replacement or revision. It occurs so much that they often get postoperative low dose radiation and NSAlDs to try as prophylactic therapy

If they show you normal SI joints - then show you anything in the spine it’s not AS. It has to hit the SI joints first (especially on multiple choice).

690
Q

In flam m a to ry B ow e l D is e a s e (E n te ro p a th ic )

overview

A

Allegedly 20% o f patient’s with Crohns & UC have a chronic inflammatory arthritis.

691
Q

In flam m a to ry B ow e l D is e a s e (E n te ro p a th ic )

2 types

A

(A): Axial Arthritis (favors SI joints and spine) - often unrelated to bowel disease
(B): Peripheral Arthritis - this one varies depending on the severity o f the bowel disease.

692
Q

S I Jo int Involvement Patterns (Rheuma toid Variants)

uniilateral

A

infection

693
Q

S I Jo int Involvement Patterns (Rheuma toid Variants)

asymmetric

A

psoriasis or reiters

694
Q

S I Jo int Involvement Patterns (Rheuma toid Variants)

symmetric

A

inflammatory bowel, AS

695
Q

Psoriatic arthritis quick

A

M=F

asymmetric SI joint

hands, feet, thoracolumbar spine

696
Q

reiters (reactive) quick

A

M>F

asymmetric SI joint

feet, lumbarspine, SI joint

697
Q

Ankylosing spondylitis quick

A

M>F

symmetric SI joint

SI joint, spine (whole thing)

698
Q

Gout

overview

A

This is a crystal arthropathy from the deposition o f uric acid crystals in and around the joints. It’s almost always in a man over 40. The big toe is the classic location.

699
Q

Gout

buzzwords/things to know

A

Earliest Sign = Joint Effusion

Spares the Joint Space (until late in the disease);
Juxtaarticular Erosions - away from the joint.

“Punched out lytic lesions”

“Overhanging Edges”

Soft tissue tophi

700
Q

Gout

MR

A
  • Juxta-articular soft tissue mass (LOW ON T2).

* The tophus will typically enhance.

701
Q

Gout

mimickers

A
“American Roentgen Ray Society Hooray”
Amyloid
RA (cystic)
Reticular Histocytosis (the most rare)
Sarcoid
Hyperlipidemia
702
Q

CPPD

overview

A

Calcium Pyrophosphate Dihydrate Disease is super common in old people. It often causes chondocalcinosis (although there are other causes). Synovitis + CPPD = “Pseudogout.” CPPD loves the triangular fibrocartilage o f the wrist, the peri-odontoid tissue, and intervertebral disks. Another important phrase is “degenerative change in an uncommon joint” - shoulder, elbow, patellofemoral joint, radiocarpal joint. Having said that, pyrophosphate arthropathy is most common at the knee.

703
Q

f you see isolated disease in the patellofemoral, radiocarpal, or talonavicular joint,

A

Think CPPD

704
Q

Classic look for CPPD

A

Hooked MCP Osteophytes with chondrocalcinosis in the TFCC is a classic took (although hemochromatosis can also look that way).

705
Q

CPPD can (and does commonly) cause

A

SLAC wrist by degenerating the SL Ligament.

706
Q

THIS vs THAT — OA vs CPPD

A

There are many overlapping features including joint space narrowing, subchondral sclerosis, subchondral cyst, and osteophyte formation. However, CPPD has some unique features, such as an “atypical joint distribution” - favoring compartments like the patellofemoral or radiocarpal. Subchondral cyst formation can be bigger than expected.

707
Q

H em o ch rom a to s is :

overview

A

This iron overload disease also is known for calcium
pyrophosphate deposition and resulting
chondrocalcinosis. It has a similar distribution to
CPPD (MCP joints). Both CPPD and
Hemochromatosis will have “hooked osteophytes”
at the MCP joint.

708
Q

H em o ch rom a to s is :

trivia

A

As a point o f trivia, therapy for the systemic disease does NOT affect the arthritis.

709
Q

THIS v s THAT CPPD an d Hemochromatosis

A

Hemochromatosis has uniform joint space loss at
ALL the MCP joints. CPPD favors the index and
middle finger MCPs.

710
Q

“M ilw a u k e e S h o u ld er”

overview

A

This is an apocalyptic destruction o f the shoulder (almost looks n e u ro p a th ic ) secondary to the demon mineral h y d ro x y ap a tite .
The articular surface changes will be very advanced, and you have a lot o f intra-articular loose bodies.

The humeral head will look like it needed the United Nations to delivery a binding resolution to keep the hydroxyapatite from fucking destroying it.

711
Q

“M ilw a u k e e S h o u ld er”

classic history

A

Old women with a history o f trauma to that joint.

712
Q

Hyperparathyroidism

overview

A

As you may remember from medical school, this can be primary or secondary, and its effects on calcium metabolism typically manifest in the bones.

Here are your buzzwords: “Subperiosteal bone resorption’ ’ o f the radial aspect o f the 2nd and 3rd fingers, rugger-jersey spine, brown tumors, and terminal tuft erosions.

713
Q

Hyperparathyroidism

classic ways to show this

A

Superior an d inferior rib notching - bone resorption

Resorption along the radial aspect o f the fin g e r s with brown tumors

Tuft Resorption

Rugger Jersey Spine

Pelvis with Narrowing or “Constricting ” o f the fem o ra l necks, an d wide S I joints.

714
Q

Arthritis problem solving

one or more

  • symmetric joint space narrowing
  • erosions
A

Inflammatory > one joint > infection

Inflammatory > multiple joints > no bony proliferation and proximal distribution > RA

Inflammatory > multiple joints > Bony proliferation and distal distribution > AS, psoriasis, reactive, inflammatory, bowel related

715
Q

Arthritis problem solving

one or more

  • asymmetric joint space narrowing
  • osteophytes
A

Degenerative > typical joints (hands, knee) > OA

Degenerative > atypical joint and age > post traumatic, gout or cppd, hemophilia

Degenerative > atypical joint and age, severe or surgical destruction > neuropathic

716
Q

Spine Degenerative Change

overview

A

In the real world it’s usually just multilevel degenerative change. But in multiple choice world you should be thinking about other things. Shiny comers with early AS, or flowing syndesmophytes with later AS. DISH with the bulky osteophytes sparing the disc space. The big bridging lateral osteophyte is classically shown for psoriatic arthritis.

717
Q

Cervical Spine:

Gamesmanship

A

Fusion: Either congenital (Klippel- Feil) or Juvenile RA.

Erosions of the Dens: CPPD and RA famously do this.

Bad Kyphosis = NF1

718
Q

DISH (Diffuse Idiopathic Skeletal H yperostosis

A

You see ossification o f the anterior longitudinal ligament involving more than 4 levels with sparing o f the disc spaces, you say DISH. The thoracic spine is most commonly used. These guys often have bony proliferation at pelvis, ischial tuberosities, trochanters, and iliac crests. There is no sacroiliitis (helps you differentiate from AS).

719
Q

OPLL (Ossification of the Posterior Longitudinal Ligament):

A

This is an ossification o f the posterior longitudinal ligament. It is associated with DISH, ossification o f the ligamentum flavum, and Ankylosing Spondylitis. It favors the cervical spine o f old Asian men. It can cause spinal canal stenosis, and can lead to cord injury after minor trauma. A key point is that it’s bad news in the cervical spine (where it is most common); in the thoracic spine it is usually asymptomatic.

720
Q

Destructive Spondyloarthropathy

A

This is associated with patients on renal dialysis (for at least 2 years), and it most commonly affects the C-spine. It looks like bad degenerative changes or CPPD. Amyloid deposition is supposedly why it happens.

721
Q

Systemic Lupus Erythematosus

arhtritis

A

The Aunt Minnie look is reducible deformity of joints without articular erosions. Joint space narrowing and erosions are uncommon findings. They can show you the hands with ulnar subluxations at the MCPs on Norgaard view, then they reduce on AP (because the hands are flat).

This ligamentous laxity also increases risk of patellar dislocations.

722
Q

Jaccoud’s Arthropathy

A

This is very similar to SLE in the hand (people often say them together). You have non erosive arthropathy with ulnar deviation of the 2nd-5th fingers at the MCP joint. The history is post rheumatic fever.

723
Q

Mixed Connective Tissue Disease

A

One unique feature is that it is positive for some antibody Ribonucleoprotein (RNP) - and therefore serology is essential to the diagnosis.

724
Q

Juvenile Idiopathic Arthritis

overview

A

This occurs before age 16 (by definition). What you see is a washed out hand that has a proximal distribution (carpals are jacked), and is ankylosed (premature fusion of growth plates). Serology is often negative (85%). In the knees, you see enlargement of the epiphyses and widened intercondylar notch - similar to findings in hemophilia.

725
Q

Juvenile Idiopathic Arthritis

buzzword

A

“Epiphyseal Overgrowth”

Widened intercondylar notch

726
Q

Amyloid Arthropathy

A

This is seen with patients on dialysis (less commonly in patients with chronic inflammation such as RA). The pattern of destruction can be severe - similar to septic arthritis or neuropathic spondyloarthropathy. The distribution is key, with bilateral involvement of the shoulders, hips, carpals, and knees being typical. Carpal tunnel syndrome is a common clinical manifestation. The joint space is typically preserved until later in the disease. When associated with dialysis, it’s rare before 5 years of treatment, but very common after 10 years (80%).

727
Q

Pituitary Gigantism

A

If they happen to show you x-rays of Andre the Giant, look for “widening of the joint space in an adult hip” - can be a classic buzzword. Late in the game, the cartilage will actually outgrow its blood supply and collapse, leading to early onset osteoarthritis. The formation of endochondral bone at existing chondro-osseous junctions results in widening of osseous structure.

728
Q

Bone marrow consists o f three components

A

(1) Trabecular Bone - the support structure, (2) Red Marrow - for making blood, and (3) Yellow Marrow - f a t for a purpose unknown at this time.

729
Q

Marrow Conversion:

A

The basic rules are that yellow marrow increases with age, in a predictable and progressive way. This is usually completed by the mid 20s. You are bom with all red marrow, and the conversion o f red to yellow occurs from the extremities to the axial skeleton (feet and hands first). Within each long bone the progression occurs epiphyses / apophyses first -> diaphysis -> followed by the distal metaphysis , and finally the proximal metaphysis. Red marrow can be found in the humeral heads and femoral heads as a normal variant in adults.

730
Q

Yellow marrow expands

A

out from the middle

731
Q

Asa child you have diffuse

A

red marrow except for ossified epiphyses and apophyses

732
Q

As adults you have yellow marrow

A

everywhere except in the axial skeleton and proximal metaphyses of proximal long bones

733
Q

What is the normal pattern o f conversion ?

A

The epiphyses convert to fatty marrow almost immediately after ossification. Distal then proceeds medial / proximal (diaphysis first, then metaphysis).

734
Q

What is the normal pattern o f Reconversion ?

A

The pattern o f reconversion: This occurs in the reverse order o f normal marrow conversion, beginning in the axial skeleton and heading peripheral. The last to go are the more distal long bones. Typically, the epiphyses are spared unless the hematopoietic demand is very high.

spine > flat bones > long bone metaphysis > long bone diaphysis > long bone epiphyses

735
Q

What areas are spared / normal variants?

A

Patchy areas o f red marrow may be seen in the proximal femoral metaphysis o f teenagers. Distal femoral sparing is seen in teenagers and menstruating women.

736
Q

Leukemia

overview

A

Proliferation o f leukemic cells results in replacement o f red marrow. Marrow will look darker than muscle (and normal disks) on T l. On STIR, marrow may be brighter than muscle because o f the increased water content. T2 is variable, often looking like diffuse red marrow.

737
Q

Leukemia

shown in 2 ways

A

(1) Lucent metaphyseal bands in a kid
(2) Tl-weighted MRI showing marrow darker than adjacent disks and muscle. Remember that Red Marrow is still 40% fat and should be brighter than muscle on T l.

738
Q

Most infdtrative conditions affect the marrow diffusely. The exceptions are

A

multiple myeloma, which has a predilection fo r focal “speckled ” deposits, and Waldenstrom’s macroglobulinemia, which causes infarcts.

739
Q

C hlo roma (G ra n u lo c y tic S a rcom a )

A

Just say “destructive mass in a bone o f a leukemia patient. ” It’s some kind o f colloid tumor.

740
Q

Anisotropy

overview

A

The most common and most problematic issues with ultrasounding tendons is this thing called “anisotropy.” The tendon is normally hyperechoic, but if you look at it when it’s NOT perpendicular to the sound beam it can look hypoechoic (injured?).

741
Q

Anisotropy

pain

A

“Supraspinatus tendon - as it curves along the contours o f the humeral head
*Long Head o f the Biceps - In the bicipital groove

742
Q

US

Tears

A

The tendon is usually hyperechoic. Focal hypoechoic areas are tears. It can be really tricky to tell if it’s partial or complete (that’s what MRI is for).

743
Q

US

Tenosynovitis

A

If they show it on ultrasound, you are looking for increased fluid within the tendon sheath. You could also see associated peritendinous subcutaneous hyperemia on Doppler.

744
Q

Tenosynovitis =

A

increased fluid in the tendon sheath

745
Q

US

Plantar Fasciitis

overview

A

This is another pathology that lends itself to a “what is it ? ” type o f ultrasound question. Hopefully, they at
least tell you this is the foot (they could label the calcaneus). The finding will be thickening o f the plantar fascia (greater than 4 mm), with loss o f the normal fibrillar pattern. If you see calipers on the plantar fascia - this is going to be the answer.

746
Q

US

Plantar Fasciitis

trivia

A

Most commonly involves the central band (there are 3 bands - people who do n ’t know anatomy think there are two).

747
Q

Calcific Tendonitis

US

A

this is very common and related to hydroxyapatite. The most common site is the supraspinatus tendon, near its insertion. It will shadow ju st like a stone in the GB.

748
Q

Calcific Tendonitis =

A

shadowing calcification in the classic locations (supraspinatus)

749
Q

Hip procedure steps

A

( l ) Mark the femoral artery. (2) Internally rotate the hip (slightly) to localize the femoral head-neck junction (your target). (3) Clean and numb the skin. (4) Advance a 20-22 gauge spinal needle into the joint - straight down on the superior head neck junction. (5) Inject a small amount o f contrast to confirm position. Contrast should flow away from the tip. If the contrast just stays there it’s not in the capsule. (6) Put the rest o f the contrast in.

750
Q

Hip procedures

trivia 1

A

Capsule is widest at the head-neck junction.

751
Q

Hip procedures

trivia 2

A

The cocktail injected is around 14 cc total (4 cc Lidocaine, 10 cc Visipaque, and only about 0.1 cc Gd).

752
Q

Shoulder procedures steps

A

(1) Supinate the hand (externally rotate the shoulder) (2) Clean and numb the skin. (3) Advance a 20-22 gauge
spinal needle into the joint - straight down on the junction between the middle and inferior third o f the humeral head - 2 mm inside the cortex. (4) Once you strike bone, pull back 1 mm and turn the bevel towards the humeral head - this should drop into the joint (5) Inject a small amount o f contrast to confirm position. Contrast should flow away from the tip. If the contrast just stays there it’s not in a space. (6) Put the rest o f the contrast in.

753
Q

Shoulders procedures trivia

A

The cocktail injected is around 12cc total (4cc Lidocaine, 8cc Visipaque, and only about 0.1 cc Gd).

754
Q

Patient is allergic to (has a phobia of) Gd, but needs an arthrogram?

A

You could try a CT Arthrogram. So - not Gad, ju st Visipaque and Lidocaine.

755
Q

Patient is allergic to (has a phobia of) CT Contrast (Visi or Omni), but needs a steroid joint injection?

A

You can inject air into the joint (instead o f visi) to confirm placement - then put the steroid in.

756
Q

Patient is allergic to (has a phobia of) CT Contrast (Visi or Omni), but needs an arthrogram ?

A

This situation is different. You c an’t inject air because you will end up with a big blooming mess on MR. CT isn’t an option either - because obviously you need the CT Contrast or it’s not an arthrogram. Your only choices would be to either (1) pre medicate them, or (2) use the force (trust your feelings…) and hope you can get in the joint without confirming positioning with fluoro.